summaryrefslogtreecommitdiff
path: root/help/LaTeX-FAQ-francaise/part2
blob: 6cec50fa9f208ef4488fe60a685a3909d176a085 (plain)
1
2
3
4
5
6
7
8
9
10
11
12
13
14
15
16
17
18
19
20
21
22
23
24
25
26
27
28
29
30
31
32
33
34
35
36
37
38
39
40
41
42
43
44
45
46
47
48
49
50
51
52
53
54
55
56
57
58
59
60
61
62
63
64
65
66
67
68
69
70
71
72
73
74
75
76
77
78
79
80
81
82
83
84
85
86
87
88
89
90
91
92
93
94
95
96
97
98
99
100
101
102
103
104
105
106
107
108
109
110
111
112
113
114
115
116
117
118
119
120
121
122
123
124
125
126
127
128
129
130
131
132
133
134
135
136
137
138
139
140
141
142
143
144
145
146
147
148
149
150
151
152
153
154
155
156
157
158
159
160
161
162
163
164
165
166
167
168
169
170
171
172
173
174
175
176
177
178
179
180
181
182
183
184
185
186
187
188
189
190
191
192
193
194
195
196
197
198
199
200
201
202
203
204
205
206
207
208
209
210
211
212
213
214
215
216
217
218
219
220
221
222
223
224
225
226
227
228
229
230
231
232
233
234
235
236
237
238
239
240
241
242
243
244
245
246
247
248
249
250
251
252
253
254
255
256
257
258
259
260
261
262
263
264
265
266
267
268
269
270
271
272
273
274
275
276
277
278
279
280
281
282
283
284
285
286
287
288
289
290
291
292
293
294
295
296
297
298
299
300
301
302
303
304
305
306
307
308
309
310
311
312
313
314
315
316
317
318
319
320
321
322
323
324
325
326
327
328
329
330
331
332
333
334
335
336
337
338
339
340
341
342
343
344
345
346
347
348
349
350
351
352
353
354
355
356
357
358
359
360
361
362
363
364
365
366
367
368
369
370
371
372
373
374
375
376
377
378
379
380
381
382
383
384
385
386
387
388
389
390
391
392
393
394
395
396
397
398
399
400
401
402
403
404
405
406
407
408
409
410
411
412
413
414
415
416
417
418
419
420
421
422
423
424
425
426
427
428
429
430
431
432
433
434
435
436
437
438
439
440
441
442
443
444
445
446
447
448
449
450
451
452
453
454
455
456
457
458
459
460
461
462
463
464
465
466
467
468
469
470
471
472
473
474
475
476
477
478
479
480
481
482
483
484
485
486
487
488
489
490
491
492
493
494
495
496
497
498
499
500
501
502
503
504
505
506
507
508
509
510
511
512
513
514
515
516
517
518
519
520
521
522
523
524
525
526
527
528
529
530
531
532
533
534
535
536
537
538
539
540
541
542
543
544
545
546
547
548
549
550
551
552
553
554
555
556
557
558
559
560
561
562
563
564
565
566
567
568
569
570
571
572
573
574
575
576
577
578
579
580
581
582
583
584
585
586
587
588
589
590
591
592
593
594
595
596
597
598
599
600
601
602
603
604
605
606
607
608
609
610
611
612
613
614
615
616
617
618
619
620
621
622
623
624
625
626
627
628
629
630
631
632
633
634
635
636
637
638
639
640
641
642
643
644
645
646
647
648
649
650
651
652
653
654
655
656
657
658
659
660
661
662
663
664
665
666
667
668
669
670
671
672
673
674
675
676
677
678
679
680
681
682
683
684
685
686
687
688
689
690
691
692
693
694
695
696
697
698
699
700
701
702
703
704
705
706
707
708
709
710
711
712
713
714
715
716
717
718
719
720
721
722
723
724
725
726
727
728
729
730
731
732
733
734
735
736
737
738
739
740
741
742
743
744
745
746
747
748
749
750
751
752
753
754
755
756
757
758
759
760
761
762
763
764
765
766
767
768
769
770
771
772
773
774
775
776
777
778
779
780
781
782
783
784
785
786
787
788
789
790
791
792
793
794
795
796
797
798
799
800
801
802
803
804
805
806
807
808
809
810
811
812
813
814
815
816
817
818
819
820
821
822
823
824
825
826
827
828
829
830
831
832
833
834
835
836
837
838
839
840
841
842
843
844
845
846
847
848
849
850
851
852
853
854
855
856
857
858
859
860
861
862
863
864
865
866
867
868
869
870
871
872
873
874
875
876
877
878
879
880
881
882
883
884
885
886
887
888
889
890
891
892
893
894
895
896
897
898
899
900
901
902
903
904
905
906
907
908
909
910
911
912
913
914
915
916
917
918
919
920
921
922
923
924
925
926
927
928
929
930
931
932
933
934
935
936
937
938
939
940
941
942
943
944
945
946
947
948
949
950
951
952
953
954
955
956
957
958
959
960
961
962
963
964
965
966
967
968
969
970
971
972
973
974
975
976
977
978
979
980
981
982
983
984
985
986
987
988
989
990
991
992
993
994
995
996
997
998
999
1000
1001
1002
1003
1004
1005
1006
1007
1008
1009
1010
1011
1012
1013
1014
1015
1016
1017
1018
1019
1020
1021
1022
1023
1024
1025
1026
1027
1028
1029
1030
1031
1032
1033
1034
1035
1036
1037
1038
1039
1040
1041
1042
1043
1044
1045
1046
1047
1048
1049
1050
1051
1052
1053
1054
1055
1056
1057
1058
1059
1060
1061
1062
1063
1064
1065
1066
1067
1068
1069
1070
1071
1072
1073
1074
1075
1076
1077
1078
1079
1080
1081
1082
1083
1084
1085
1086
1087
1088
1089
1090
1091
1092
1093
1094
1095
1096
1097
1098
1099
1100
1101
1102
1103
1104
1105
1106
1107
1108
1109
1110
1111
1112
1113
1114
1115
1116
1117
1118
1119
1120
1121
1122
1123
1124
1125
1126
1127
1128
1129
1130
1131
1132
1133
1134
1135
1136
1137
1138
1139
1140
1141
1142
1143
1144
1145
1146
1147
1148
1149
1150
1151
1152
1153
1154
1155
1156
1157
1158
1159
1160
1161
1162
1163
1164
1165
1166
1167
1168
1169
1170
1171
1172
1173
1174
1175
1176
1177
1178
1179
1180
1181
1182
1183
1184
1185
1186
1187
1188
1189
1190
1191
1192
1193
1194
1195
1196
1197
1198
1199
1200
1201
1202
1203
1204
1205
1206
1207
1208
1209
1210
1211
1212
1213
1214
1215
1216
1217
1218
1219
1220
1221
1222
1223
1224
1225
1226
1227
1228
1229
1230
1231
1232
1233
1234
1235
1236
1237
1238
1239
1240
1241
1242
1243
1244
1245
1246
1247
1248
1249
1250
1251
1252
1253
1254
1255
1256
1257
1258
1259
1260
1261
1262
1263
1264
1265
1266
1267
1268
1269
1270
1271
1272
1273
1274
1275
1276
1277
1278
1279
1280
1281
1282
1283
1284
1285
1286
1287
1288
1289
1290
1291
1292
1293
1294
1295
1296
1297
1298
1299
1300
1301
1302
1303
1304
1305
1306
1307
1308
1309
1310
1311
1312
1313
1314
1315
1316
1317
1318
1319
1320
1321
1322
1323
1324
1325
1326
1327
1328
1329
1330
1331
1332
1333
1334
1335
1336
1337
1338
1339
1340
1341
1342
1343
1344
1345
1346
1347
1348
1349
1350
1351
1352
1353
1354
1355
1356
1357
1358
1359
1360
1361
1362
1363
1364
1365
1366
1367
1368
1369
1370
1371
1372
1373
1374
1375
1376
1377
1378
1379
1380
1381
1382
1383
1384
1385
1386
1387
1388
1389
1390
1391
1392
1393
1394
1395
1396
1397
1398
1399
1400
1401
1402
1403
1404
1405
1406
1407
1408
1409
1410
1411
1412
1413
1414
1415
1416
1417
1418
1419
1420
1421
1422
1423
1424
1425
1426
1427
1428
1429
1430
1431
1432
1433
1434
1435
1436
1437
1438
1439
1440
1441
1442
1443
1444
1445
1446
1447
1448
1449
1450
1451
1452
1453
1454
1455
1456
1457
1458
1459
1460
1461
1462
1463
1464
1465
1466
1467
1468
1469
1470
1471
1472
1473
1474
1475
1476
1477
1478
1479
1480
1481
1482
1483
1484
1485
1486
1487
1488
1489
1490
1491
1492
1493
1494
1495
1496
1497
1498
1499
1500
1501
1502
1503
1504
1505
1506
1507
1508
1509
1510
1511
1512
1513
1514
1515
1516
1517
1518
1519
1520
1521
1522
1523
1524
1525
1526
1527
1528
1529
1530
1531
1532
1533
1534
1535
1536
1537
1538
1539
1540
1541
1542
1543
1544
1545
1546
1547
1548
1549
1550
1551
1552
1553
1554
1555
1556
1557
1558
1559
1560
1561
1562
1563
1564
1565
1566
1567
1568
1569
1570
1571
1572
1573
1574
1575
1576
1577
1578
1579
1580
1581
1582
1583
1584
1585
1586
1587
1588
1589
1590
1591
1592
1593
1594
1595
1596
1597
1598
1599
1600
1601
1602
1603
1604
1605
1606
1607
1608
1609
1610
1611
1612
1613
1614
1615
1616
1617
1618
1619
1620
1621
1622
1623
1624
1625
1626
1627
1628
1629
1630
1631
1632
1633
1634
1635
1636
1637
1638
1639
1640
1641
1642
1643
1644
1645
1646
1647
1648
1649
1650
1651
1652
1653
1654
1655
1656
1657
1658
1659
1660
1661
1662
1663
1664
1665
1666
1667
1668
1669
1670
1671
1672
1673
1674
1675
1676
1677
1678
1679
1680
1681
1682
1683
1684
1685
1686
1687
1688
1689
1690
1691
1692
1693
1694
1695
1696
1697
1698
1699
1700
1701
1702
1703
1704
1705
1706
1707
1708
1709
1710
1711
1712
1713
1714
1715
1716
1717
1718
1719
1720
1721
1722
1723
1724
1725
1726
1727
1728
1729
1730
1731
1732
1733
1734
1735
1736
1737
1738
1739
1740
1741
1742
1743
1744
1745
1746
1747
1748
1749
1750
1751
1752
1753
1754
1755
1756
1757
1758
1759
1760
1761
1762
1763
1764
1765
1766
1767
1768
1769
1770
1771
1772
1773
1774
1775
1776
1777
1778
1779
1780
1781
1782
1783
1784
1785
1786
1787
1788
1789
1790
1791
1792
1793
1794
1795
1796
1797
1798
1799
1800
1801
1802
1803
1804
1805
1806
1807
1808
1809
1810
1811
1812
1813
1814
1815
1816
1817
1818
1819
1820
1821
1822
1823
1824
1825
1826
1827
1828
1829
1830
1831
1832
1833
1834
1835
1836
1837
1838
1839
1840
1841
1842
1843
1844
1845
1846
1847
1848
1849
1850
1851
1852
1853
1854
1855
1856
1857
1858
1859
1860
1861
1862
1863
1864
1865
1866
1867
1868
1869
1870
1871
1872
1873
1874
1875
1876
1877
1878
1879
1880
1881
1882
1883
1884
1885
1886
1887
1888
1889
1890
1891
1892
1893
1894
1895
1896
1897
1898
1899
1900
1901
1902
1903
1904
1905
1906
1907
1908
1909
1910
1911
1912
1913
1914
1915
1916
1917
1918
1919
1920
1921
1922
1923
1924
1925
1926
1927
1928
1929
1930
1931
1932
1933
1934
1935
1936
1937
1938
1939
1940
1941
1942
1943
1944
1945
1946
1947
1948
1949
1950
1951
1952
1953
1954
1955
1956
1957
1958
1959
1960
1961
1962
1963
1964
1965
1966
1967
1968
1969
1970
1971
1972
1973
1974
1975
1976
1977
1978
1979
1980
1981
1982
1983
1984
1985
1986
1987
1988
1989
1990
1991
1992
1993
1994
1995
1996
1997
1998
1999
2000
2001
2002
2003
2004
2005
2006
2007
2008
2009
2010
2011
2012
2013
2014
2015
2016
2017
2018
2019
2020
2021
2022
2023
2024
2025
2026
2027
2028
2029
2030
2031
2032
2033
2034
2035
2036
2037
2038
2039
2040
2041
2042
2043
2044
2045
2046
2047
2048
2049
2050
2051
2052
2053
2054
2055
2056
2057
2058
2059
2060
2061
2062
2063
2064
2065
2066
2067
2068
2069
2070
2071
2072
2073
2074
2075
2076
2077
2078
2079
2080
2081
2082
2083
2084
2085
2086
2087
2088
2089
2090
2091
2092
2093
2094
2095
2096
2097
2098
2099
2100
2101
2102
2103
2104
2105
2106
2107
2108
2109
2110
2111
2112
2113
2114
2115
2116
2117
2118
2119
2120
2121
2122
2123
2124
2125
2126
2127
2128
2129
2130
2131
2132
2133
2134
2135
2136
2137
2138
2139
2140
2141
2142
2143
2144
2145
2146
2147
2148
2149
2150
2151
2152
2153
2154
2155
2156
2157
2158
2159
2160
2161
2162
2163
2164
2165
2166
2167
2168
2169
2170
2171
2172
2173
2174
2175
2176
2177
2178
2179
2180
2181
2182
2183
2184
2185
2186
2187
2188
2189
2190
2191
2192
2193
2194
2195
2196
2197
2198
2199
2200
2201
2202
2203
2204
2205
2206
2207
2208
2209
2210
2211
2212
2213
2214
2215
2216
2217
2218
2219
2220
2221
2222
2223
2224
2225
2226
2227
2228
2229
2230
2231
2232
2233
2234
2235
2236
2237
2238
2239
2240
2241
2242
2243
2244
2245
2246
2247
2248
2249
2250
2251
2252
2253
2254
2255
2256
2257
2258
2259
2260
2261
2262
2263
2264
2265
2266
2267
2268
2269
2270
2271
2272
2273
2274
2275
2276
2277
2278
2279
2280
2281
2282
2283
2284
2285
2286
2287
2288
2289
2290
2291
2292
2293
2294
2295
2296
2297
2298
2299
2300
2301
2302
2303
2304
2305
2306
2307
2308
2309
2310
2311
2312
2313
2314
2315
2316
2317
2318
2319
2320
2321
2322
2323
2324
2325
2326
2327
2328
2329
2330
2331
2332
2333
2334
2335
2336
2337
2338
2339
2340
2341
2342
2343
2344
2345
2346
2347
2348
2349
2350
2351
2352
2353
2354
2355
2356
2357
2358
2359
2360
2361
2362
2363
2364
2365
2366
2367
2368
2369
2370
2371
2372
2373
2374
2375
2376
2377
2378
2379
2380
2381
2382
2383
2384
2385
2386
2387
2388
2389
2390
2391
2392
2393
2394
2395
2396
2397
2398
2399
2400
2401
2402
2403
2404
2405
2406
2407
2408
2409
2410
2411
2412
2413
2414
2415
2416
2417
2418
2419
2420
2421
2422
2423
2424
2425
2426
2427
2428
2429
2430
2431
2432
2433
2434
2435
2436
2437
2438
2439
2440
2441
2442
2443
2444
2445
2446
2447
2448
2449
2450
2451
2452
2453
2454
2455
2456
2457
2458
2459
2460
2461
2462
2463
2464
2465
2466
2467
2468
2469
2470
2471
2472
2473
2474
2475
2476
2477
2478
2479
2480
2481
2482
2483
2484
2485
2486
2487
2488
2489
2490
2491
2492
2493
2494
2495
2496
2497
2498
2499
2500
2501
2502
2503
2504
2505
2506
2507
2508
2509
2510
2511
2512
2513
2514
2515
2516
2517
2518
2519
2520
2521
2522
2523
2524
2525
2526
2527
2528
2529
2530
2531
2532
2533
2534
2535
2536
2537
2538
2539
2540
2541
2542
2543
2544
2545
2546
2547
2548
2549
2550
2551
2552
2553
2554
2555
2556
2557
2558
2559
2560
2561
2562
2563
2564
2565
2566
2567
2568
2569
2570
2571
2572
2573
2574
2575
2576
2577
2578
2579
2580
2581
2582
2583
2584
2585
2586
2587
2588
2589
2590
2591
2592
2593
2594
2595
2596
2597
2598
2599
2600
2601
2602
2603
2604
2605
2606
2607
2608
2609
2610
2611
2612
2613
2614
2615
2616
2617
2618
2619
2620
2621
2622
2623
2624
2625
2626
2627
2628
2629
2630
2631
2632
2633
2634
2635
2636
2637
2638
2639
2640
2641
2642
2643
2644
2645
2646
2647
2648
2649
2650
2651
2652
2653
2654
2655
2656
2657
2658
2659
2660
2661
2662
2663
2664
2665
2666
2667
2668
2669
2670
2671
2672
2673
2674
2675
2676
2677
2678
2679
2680
2681
2682
2683
2684
2685
2686
2687
2688
2689
2690
2691
2692
2693
2694
2695
2696
2697
2698
2699
2700
2701
2702
2703
2704
2705
2706
2707
2708
2709
2710
2711
2712
2713
2714
2715
2716
2717
2718
2719
2720
2721
2722
2723
2724
2725
2726
2727
2728
2729
2730
2731
2732
2733
2734
2735
2736
2737
2738
2739
2740
2741
2742
2743
2744
2745
2746
2747
2748
2749
2750
2751
2752
2753
2754
2755
2756
2757
2758
2759
2760
2761
2762
2763
2764
2765
2766
2767
2768
2769
2770
2771
2772
2773
2774
2775
2776
2777
2778
2779
2780
2781
2782
2783
2784
2785
2786
2787
2788
2789
2790
2791
2792
2793
2794
2795
2796
2797
2798
2799
2800
2801
2802
2803
2804
2805
2806
2807
2808
2809
2810
2811
2812
2813
2814
2815
2816
2817
2818
2819
2820
2821
2822
2823
2824
2825
2826
2827
2828
2829
2830
2831
2832
2833
2834
2835
2836
2837
2838
2839
2840
2841
2842
2843
2844
2845
2846
2847
2848
2849
2850
2851
2852
2853
2854
2855
2856
2857
2858
2859
2860
2861
2862
2863
2864
2865
2866
2867
2868
2869
2870
2871
2872
2873
2874
2875
2876
2877
2878
2879
2880
2881
2882
2883
2884
2885
2886
2887
2888
2889
2890
2891
2892
2893
2894
2895
2896
2897
2898
2899
2900
2901
2902
2903
2904
2905
2906
2907
2908
2909
2910
2911
2912
2913
2914
2915
2916
2917
2918
2919
2920
2921
2922
2923
2924
2925
2926
2927
2928
2929
2930
2931
2932
2933
2934
2935
2936
2937
2938
2939
2940
2941
2942
2943
2944
2945
2946
2947
2948
2949
2950
2951
2952
2953
2954
2955
2956
Path: tempo.univ-lyon1.fr!univ-lyon1.fr!howland.erols.net!dispose.news.demon.net!demon!grolier!fr.usenet-edu.net!usenet-edu.net!teaser.fr!fdn.fr!edgard.fdn.fr!not-for-mail
Date: 08 Jun 2001 13:31:17 +0200
Newsgroups: fr.comp.text.tex,fr.usenet.reponses
Subject: [FAQ] fr.comp.text.tex - partie  6
Message-ID: <faq-fctt-p220010608133117@edgard.fdn.fr>
X-Posted-By: poste.sh version 1.1
From: bayartb@edgard.fdn.fr (Nono le robot)
Supersedes: <faq-fctt-p220010501143416@edgard.fdn.fr>
Expires: 08 Jul 2001 13:31:17 +0200
Followup-To: poster
Organization: French Data Network
Approved: bayartb@edgard.fdn.fr
Lines: 2947
Xref: tempo.univ-lyon1.fr fr.comp.text.tex:23525 fr.usenet.reponses:20391

Archive-name: fr/faq-latex-francaise/part2

Author: Marie-Paule Kluth
Posting-Frequency: mensuel (monthly)
Version: 2.27

================================================================
  Cette FAQ, rédigée initialement par MP Kluth est maintenant
       tenue à jour autant que possible par B. Bayart et 
          plusieurs volontaires (voir question [30]).
================================================================

================================================================
[6] GESTION DE LA MISE EN PAGE
================================================================

# 6.1 # Comment modifier l'interligne d'un document ?
-----------------------------------------------------
* Pour modifier l'espace interligne d'un document on peut
utiliser la commande \linespread (solution non recommandée).
Par exemple, \linespread{1.6} permet de doubler l'intervalle
par défaut.

* \renewcommand{\baselinestretch}{1.2} placé dans le
préambule permet d'obtenir le même résultat pour tout le
document.

* Il existe également les packages 'doublespace' (pour
LaTeX2.09) et 'setspace' (pour LaTeX2e) disponibles sur
ftp://ftp.fdn.org/pub/CTAN/macros/latex2.09/contrib/misc/ pour l'un et
ftp://ftp.fdn.org/pub/CTAN/macros/latex/contrib/other/misc/ ou
ftp://ftp.fdn.org/pub/CTAN/macros/latex/contrib/supported/setspace/ pour l'autre.
setspace.sty définit les environnements singlespace,
onehalfspace et doublespace. L'utilisation de ces styles est
recommandée parce que plus robuste (gestion des tableaux, des
notes de bas de page, ...).

* Pour réduire l'interligne d'un paragraphe (à celui de small
par exemple) sans modifier la taille de la fonte on peut
utiliser \small{\normalsize texte à interligne réduit}\par ou
encore {\advance\baselineskip -1pt le texte \par}.

* Localement, on peut également utiliser la commande
\baselineskip de la manière suivante :

Exemple :
{\setlength{\baselineskip}{1.2\baselineskip}
Texte affecté
\par} %%% <= terminer le paragraphe
%%%% fin exemple %%%%

# 6.2 # Comment gérer un document recto-verso ?
-----------------------------------------------
* LaTeX2e prévoit directement les options de classe twoside et
openright.

Exemple :
\documentclass[twoside,openright]{report}
%%%% fin exemple %%%%

* En LaTeX 2.09 il faut passer twoside comme option de la
commande documentstyle. Ensuite, pour forcer les entêtes de
chapitre à commencer sur une page impaire, il faut inclure la
commande \cleardoublepage avant chaque début de chapitre.

Exemple :
\documentstyle[twoside]{report}

\begin{document}
\cleardoublepage
\chapter{Introduction.}
	Texte.

\cleardoublepage
\chapter{Thèse.}
	Texte.

\end{document}
%%%% fin exemple %%%%

* Il existe également la macro suivante qui redéfinit la
commande \cleardoublepage pour que les pages insérées soient
vides (i.e. sans entête ni bas de page).
%%%% debut macro %%%%
% whitecdp (formerly schulzrinne.sty) --provide for blank pages
% between chapters
% This redefinition of the \cleardoublepage command provides
% for a special pagestyle for the "extra" pages which are generated
% to ensure that the chapter opener is on a recto page.
% The pagestyle is "chapterverso"; for many publishers, this should be
% identical to "empty", so that's the default.
\def\cleardoublepage{\clearpage
 \if@twoside
  \ifodd\c@page\else
   \null\thispagestyle{chapterverso}\newpage
   \if@twocolumn\null\newpage\fi
   \fi
  \fi
 }%
\def\ps@chapterverso{\ps@empty}%
%%%% fin macro %%%%

# 6.3 # Comment modifier le style des titres ?
----------------------------------------------
* Les définitions de \section, \sub(sub)section, etc, se
trouvent dans les fichiers .cls (report.cls, article.cls,
book.cls).

ATTENTION : Il est vivement conseillé de ne pas modifier
            directement ces classes mais de redéfinir
	    un fichier.sty avec les nouvelles commandes ou
	    d'utiliser \makeatletter et \makeatother.

Exemple :
La syntaxe de définition d'une nouvelle section est :
\renewcommand\section{\@startsection {section}{1}{\z@}%
	{-3.5ex \@plus -1ex \@minus -.2ex}%
	{2.3ex \@plus.2ex}%
	{\reset@font\Large\bfseries}}

Explication :
+ La commande \@startsection permet de gérer : la table des
	matières, la numérotation des titres, les références,
	les titres des sections dans l'en-tête, etc...
+ {section} indique qu'il s'agit d'une section
+ {1} indique son niveau dans la table des matières
+ {\z@} indique son niveau d'indentation (zéro)
+ {-3.5ex \@plus -1ex \@minus -.2ex} définit l'espace qui sera
	ajouté au dessus du titre
+ {2.3ex \@plus.2ex} définit l'espace qui sera ajouté en dessous
	du titre. Si ce nombre est négatif alors il s'agit d'un
	espacement horizontal, pour avoir des titres "en ligne",
	comme ça :
	Titre de ma section.        Nous allons parler... bla,
	bla, bla...
+ \@plus et \@minus permettent de jouer sur l'élasticité de ces
	espaces
+ {\reset@font\Large\bfseries} sont les commandes de mises en
	forme du titre.
%%%% fin exemple %%%%

* Pour augmenter l'espace avant une section il suffit, par
exemple, d'écrire, dans le préambule du document :

%%%% debut macro %%%%
\makeatletter
\renewcommand\section{\@startsection{section}{1}{\z@}%
	{2cm \@plus -1ex \@minus -.2ex}%
	{2.3ex \@plus.2ex}%
	{\reset@font\Large\bfseries}}
\makeatother
%%%% fin macro %%%%

* Le package 'sfheaders' de M. Loreti, ci dessous met les titres
dans une fonte sans serif quelle que soit la classe de
document utilisée. Pour l'utiliser, il suffit de sauvegarder les
macros suivantes dans un fichier SFheaders.sty et d'appeler
\usepackage{SFheaders}.

%%%% debut macro %%%%
% Package: SFheaders.sty
% Sans-Serif headers; modified from {book|report|article}.cls
% defaults.
% Last modified: MLO 1997-05-06
%
% Author: Maurizio Loreti, aka MLO or (HAM) I3NOO
% Work:   University of Padova - Department of Physics
%         Via F. Marzolo, 8 - 35131 PADOVA - Italy
% EMail:  loreti@padova.infn.it
% WWW:    http://wwwcdf.pd.infn.it/~loreti/mlo.html

\NeedsTeXFormat{LaTeX2e}
\ProvidesPackage{sfheaders}[1997/05/06 Sans-Serif headers]

\@ifclassloaded{article}{
% Here if \documentclass{article}
  \def\@part[#1]#2{%
      \ifnum \c@secnumdepth >\m@ne
        \refstepcounter{part}%
        \addcontentsline{toc}{part}{\thepart\hspace{1em}#1}%
      \else
        \addcontentsline{toc}{part}{#1}%
      \fi
      {\parindent \z@ \raggedright
       \interlinepenalty \@M
       \normalfont
       \ifnum \c@secnumdepth >\m@ne
         \Large \sffamily \bfseries \partname~\thepart
         \par\nobreak
       \fi
       \huge \sffamily \bfseries #2%
       \markboth{}{}\par}%
      \nobreak
      \vskip 3ex
      \@afterheading}
  \def\@spart#1{%
      {\parindent \z@ \raggedright
       \interlinepenalty \@M
       \normalfont
       \huge \sffamily \bfseries #1\par}%
       \nobreak
       \vskip 3ex
       \@afterheading}}
{\@ifclassloaded{book}{
% Here if \documentclass{book}
  \def\@part[#1]#2{%
      \ifnum \c@secnumdepth >-2\relax
        \refstepcounter{part}%
        \addcontentsline{toc}{part}{\thepart\hspace{1em}#1}%
      \else
        \addcontentsline{toc}{part}{#1}%
      \fi
      \markboth{}{}%
      {\centering
       \interlinepenalty \@M
       \normalfont
       \ifnum \c@secnumdepth >-2\relax
         \huge \sffamily \bfseries \partname~\thepart
         \par
         \vskip 20\p@
       \fi
       \Huge \sffamily \bfseries #2\par}%
      \@endpart}

  \def\@spart#1{%
      {\centering
       \interlinepenalty \@M
       \normalfont
       \Huge \sffamily \bfseries #1\par}%
      \@endpart}

  \def\@makechapterhead#1{%
    \vspace*{50\p@}%
    {\parindent \z@ \raggedright \normalfont
      \ifnum \c@secnumdepth >\m@ne
        \if@mainmatter
          \huge \sffamily \bfseries \@chapapp\space \thechapter
          \par\nobreak
          \vskip 20\p@
        \fi
      \fi
      \interlinepenalty\@M
      \Huge \sffamily\ bfseries #1\par\nobreak
      \vskip 40\p@
    }}

  \def\@makeschapterhead#1{%
    \vspace*{50\p@}%
    {\parindent \z@ \raggedright
      \normalfont
      \interlinepenalty\@M
      \Huge \sffamily \bfseries #1\par\nobreak
      \vskip 40\p@
  }}
}{
% Here if none of the above (\documentclass{report} ?)
  \def\@part[#1]#2{%
      \ifnum \c@secnumdepth >-2\relax
        \refstepcounter{part}%
        \addcontentsline{toc}{part}{\thepart\hspace{1em}#1}%
      \else
        \addcontentsline{toc}{part}{#1}%
      \fi
      \markboth{}{}%
      {\centering
       \interlinepenalty \@M
       \normalfont
       \ifnum \c@secnumdepth >-2\relax
         \huge \sffamily \bfseries \partname~\thepart
         \par
         \vskip 20\p@
       \fi
       \Huge \sffamily \bfseries #2\par}%
      \@endpart}

  \def\@spart#1{%
      {\centering
       \interlinepenalty \@M
       \normalfont
       \Huge \sffamily \bfseries #1\par}%
      \@endpart}

  \def\@makechapterhead#1{%
    \vspace*{50\p@}%
    {\parindent \z@ \raggedright \normalfont
      \ifnum \c@secnumdepth >\m@ne
          \huge \sffamily \bfseries \@chapapp\space \thechapter
          \par\nobreak
          \vskip 20\p@
      \fi
      \interlinepenalty\@M
      \Huge \sffamily \bfseries #1\par\nobreak
      \vskip 40\p@
    }}

  \def\@makeschapterhead#1{%
    \vspace*{50\p@}%
    {\parindent \z@ \raggedright
      \normalfont
      \interlinepenalty\@M
      \Huge \sffamily \bfseries #1\par\nobreak
      \vskip 40\p@
    }}
}}

\renewcommand{\section}{\@startsection {section}{1}{\z@}%
             {-3.5ex \@plus -1ex \@minus -.2ex}%
             {2.3ex \@plus.2ex}%
             {\normalfont\Large\sffamily\bfseries}}

\renewcommand{\subsection}{\@startsection{subsection}{2}{\z@}%
             {-3.25ex\@plus -1ex \@minus -.2ex}%
             {1.5ex \@plus .2ex}%
             {\normalfont\large\sffamily\bfseries}}

\renewcommand{\subsubsection}{\@startsection{subsubsection}{3}%
	     {\z@}%
	     {-3.25ex\@plus -1ex \@minus -.2ex}%
             {1.5ex \@plus .2ex}%
             {\normalfont\normalsize\sffamily\bfseries}}

\renewcommand{\paragraph}{\@startsection{paragraph}{4}{\z@}%
             {3.25ex \@plus1ex \@minus.2ex}%
             {-1em}%
             {normalfont\normalsize\sffamily\bfseries}}

\renewcommand{\subparagraph}{\@startsection{subparagraph}{5}%
	     {\parindent}%
             {3.25ex \@plus1ex \@minus .2ex}%
             {-1em}%
             {\normalfont\normalsize\sffamily\bfseries}}
\endinput

%%
%% End of `SFheaders.sty'.
%%%% fin macro %%%%

* Le package 'fncychap' disponible sur
ftp://ftp.fdn.org/pub/CTAN/macros/latex/contrib/supported/fncychap/ propose un
ensemble d'entêtes de chapitre prédéfinies.

# 6.4 # Comment obtenir un document multicolonnes ?
---------------------------------------------------
* L'option standard twocolumn permet de présenter un texte sur
deux colonnes verticales.

Exemple :
\documentclass[twocolumn]{article}
\usepackage{french}

\begin{document}

Voici un texte sur deux colonnes que \LaTeX n'équilibre pas
par lui-même (il remplit les colonnes les unes après les
autres). L'espace entre les colonnes peut être modifié comme
indiqué plus loin. Une ligne de séparation des colonnes peut
également être insérée.

\end{document}
%%%% fin exemple %%%%

* Pour agir localement, on peut utiliser les commandes :
\twocolumn[texte sur une colonne]{texte sur deux colonnes}
puis \onecolumn{Texte sur une colonne} ou plus généralement
\twocolumn et \onecolumn.

Pour une meilleure lisibilité du source, on peut également
utiliser les environnements correspondants.

Exemple :
\documentclass{article}
\usepackage{french}

\begin{document}

Dans la classe article, le texte est, par défaut, mis en page
sur une seule colonne. Il est toutefois possible de passer
temporairement sur deux colonnes.

\twocolumn[Un titre sur une colonne, un peu long pour le
prouver.]{Et voici enfin un texte sur deux colonnes~; comme
promis~! Encore une fois, \LaTeX n'équilibre pas le
remplissage des deux colonnes si bien qu'il est obligé pour
chaque changement de colonnage de changer de page.}

\onecolumn
Ceci permet de repasser sur une colonne pour la suite du
document.

\begin{twocolumn}
   Encore quelques mots sur deux colonnes. Même si le texte est
   trop court pour voir apparaître la deuxième colonne.
\end{twocolumn}
\end{document}
%%%% fin exemple %%%%

* Le package 'multicol', disponible sur
ftp://ftp.fdn.org/pub/CTAN/macros/latex/packages/tools/, définit l'environnement
multicols qui permet de redéfinir localement le nombre de
colonnes désirées (10 maximum). Lorsqu'une page n'est pas
complète, le texte apparaît réparti sur toutes les colonnes.
Ainsi chaque changement de colonnage n'entraîne plus un
changement de page.

Exemple :
\documentclass{article}
\usepackage{multicol}
\usepackage{french}
\setlength{\columnseprule}{0.5pt}
\begin{document}

\begin{multicols}{3}[Titre sur une seule colonne.]
   3~colonnes équilibrées, 3~colonnes équilibrées, 3~colonnes
   équilibrées, 3~colonnes équilibrées
\end{multicols}

\begin{multicols}{2}[\section{Titre numéroté.}]
   blabla sur deux colonnes, c'est plus sérieux. C'est le
   style qui est généralement utilisé pour écrire des
   articles.
\end{multicols}
\end{document}
%%%% fin exemple %%%%

Pour ajouter un titre numéroté qui apparaisse sur toute la
largeur de la page, il faut utiliser l'option [\section{Titre.}]
juste après \begin{multicols}{nb-col}.

Remarques :
+ Pour qu'une ligne de séparation apparaisse entre les
colonnes, il faut utiliser : \setlength{\columnseprule}{1pt}.

+ Pour redéfinir la largeur de l'espace inter-colonnes, il faut
utiliser \setlength{\columnsep}{30pt}.

# 6.5 # Comment composer une brochure ?
---------------------------------------
* Pour redéfinir un format de page, (par exemple un A4 plié en
trois), il faut utiliser la commande \setlength.  Il suffit de
savoir quelles sont les longueurs à préciser, le meilleur moyen
pour ce faire est de les visualiser avec la commande \layout
(définie par le package 'layout' disponible sur
ftp://ftp.fdn.org/pub/CTAN/macros/latex/packages/tools/).

%%%% debut macro %%%%
% (Th. Bouche)
\ProvidesPackage{a6size}
% rien a voir avec la taille : ajustement du \baselineskip
\renewcommand\normalsize{%
   \@setfontsize\normalsize\@xiipt{13.5}%
   \abovedisplayskip 12\p@ \@plus3\p@ \@minus7\p@
   \abovedisplayshortskip \z@ \@plus3\p@
   \belowdisplayshortskip 6.5\p@ \@plus3.5\p@ \@minus3\p@
   \belowdisplayskip \abovedisplayskip
   \let\@listi\@listI}
\renewcommand\small{%
   \@setfontsize\small\@xipt{12.4}%
   \abovedisplayskip 11\p@ \@plus3\p@ \@minus6\p@
   \abovedisplayshortskip \z@ \@plus3\p@
   \belowdisplayshortskip 6.5\p@ \@plus3.5\p@ \@minus3\p@
   \def\@listi{\leftmargin\leftmargini
   \topsep 9\p@ \@plus3\p@ \@minus5\p@
   \parsep 4.5\p@ \@plus2\p@ \@minus\p@
   \itemsep \parsep}%
   \belowdisplayskip \abovedisplayskip}
\normalsize

\setlength\paperheight {148mm}%
\setlength\paperwidth  {105mm}%
%\voffset-1cm
%\hoffset-2cm
\setlength{\topmargin}{-1.3cm}%
\setlength{\oddsidemargin}{-.5cm}%
\setlength{\evensidemargin}{-1cm}%
\setlength{\marginparsep}{0\p@}%
\setlength{\headsep}{0\p@}%
% calcule la hauteur du texte en fonction du \baselineskip, pour
% que les lignes soient placées au même niveau sur toutes les pages
\setlength{\textheight}{\topskip}
\addtolength{\textheight}{22\baselineskip}%
\setlength{\textwidth}{7cm}%
\setlength{\footskip}{23\p@}% (originellement : 48)
%\setlength{\baselineskip}{13\p@}%
%\setlength{\marginparwidth}{0\p@} %
%\addtolength{\baselineskip}{.2\baselineskip}%
\setlength{\parindent}{0\p@}
%\addtolength{\headsep}{\headsep}
%\setlength{\push@skip}{.2\textwidth}
\newenvironment{page}{\vspace*{\stretch{1}}}
{\vspace*{\stretch{2.5}}\newpage}
\pagestyle{plain}
%%%% fin macro %%%%

Il faut ensuite opérer ce que les imprimeurs appellent une
imposition : imprimer la page tant à tel endroit de la x-ième
feuille de telle sorte qu'il n'y ait plus qu'à plier la liasse
pour obtenir un livre prêt à être relié. Cette étape est facile
à réaliser soit à l'aide de dvidvi (mais qui n'autorise pas les
rotations, ce qui peut en limiter l'intérêt pour des formats
spéciaux) soit avec pstops :
pstops "2:0L@.7(21cm,0)+1L@.7(21cm,14.85cm)" un.ps deux.ps
disponible sur ftp://ftp.fdn.org/pub/CTAN/support/psutils/.

* L'option a5paper de LaTeX2e permet également de faire cela.
Il faut ensuite utiliser dvidvi et dvips en -t landscape. dvips
est disponible entre autres par ftp sur
ftp://ftp.gutenberg.eu.org/pub/gut/sources/.

* Le package 'a5booklet' est disponible sur
ftp://ftp.fdn.org/pub/CTAN/dviware/a5booklet/.

* Consulter également le package '2up' pour LaTeX2e.

* Le package 'poligraf' disponible sur
ftp://ftp.fdn.org/pub/CTAN/macros/TeX-PS/cmyk-hax/ permet d'agir sur la mise en page
d'un document avant impression.

* psnup et psbook peuvent également permettre de faire de la
composition mais il vaut alors mieux travailler avec des polices
postscript. Ces utilitaires sont accessibles via :
http://www.dcs.ed.ac.uk/home/ajcd/psutils/.

Exemple (J.A. Ferrez) :
=================SNIP
#!/bin/sh
#
# Turn a (clean) PS file into a booklet
#
# input on stdin or file in arg
#
# psbook -- reorder the pages
#           see -s option for _very_ large files
# psnup  -- scale and place two pages onto one
# pstops -- reverse the odd pages for duplex printing
#
# outup on stdout
#
psbook $1 | psnup -2 | pstops "2:0,1U(21cm,29.7cm)"
=================SNIP
%%%% fin exemple %%%%

* Le résultat est possible également en ne travaillant que sur
le postscript : dvips -h twoup -t landscape fichier.dvi.

* Sur PC on peut utiliser dvidrv.

* Pour PC twoup fait cela, mais ce n'est pas du domaine public.

# 6.6 # Comment définir un format de document ?
-----------------------------------------------
La structure d'une page LaTeX permet de nombreux ajustements :
+ \setlength{\textwidth}{??cm} permet de fixer la largeur du
texte
+ \setlength{\textheight}{??cm} permet de fixer la hauteur du
texte
+ \setlength{\oddsidemargin}{(-)??cm} permet de définir la
marge gauche des pages impaires
+ \setlength{\evensidemargin}{(-)??cm} permet de définir la
marge gauche des pages paires
+ \setlength{\topskip}{??cm} laisse un espace en haut de page
+ \setlength{\footskip}{??cm} laisse un espace en bas de page
+ \setlength{\headheight}{??cm} fixe la hauteur de l'entête
+ \addtolength{\topmargin}{(-)??cm}
+ \addtolength{\textheight}{(-)??cm}

* Il existe également les packages 'a4' et 'a4wide' disponibles
respectivement sur
ftp://ftp.fdn.org/pub/CTAN/macros/latex/contrib/supported/ntgclass/ et dans
ftp://ftp.fdn.org/pub/CTAN/macros/latex/contrib/other/misc/ qui permettent de
redéfinir les marges et largeur de texte.

* Le package 'geometry' est plus flexible que les précédents
pour définir son propre format de page. Il définit de
nouvelles variables de structure de la page. Il propose en
outre un ensemble de formats par défaut (a0paper, b5paper,
legalpaper, ...).

# 6.7 # Comment redéfinir les marges d'un document ?
----------------------------------------------------
* Le package 'vmargin', de V. Kuhlmann, est disponible sur
ftp://ftp.fdn.org/pub/CTAN/macros/latex/contrib/other/misc/. Il  permet facilement de
redéfinir globalement (pour tout le document) les marges d'un
document par la commande :
\setmarginsrb{1}{2}{3}{4}{5}{6}{7}{8}
1 est la marge gauche
2 est la marge en haut
3 est la marge droite
4 est la marge en bas
5 fixe la hauteur de l'entête
6 fixe la distance entre l'entête et le texte
7 fixe la hauteur du pied de page
8 fixe la distance entre le texte et le pied de page

Vous pouvez également utiliser des valeurs par défaut en rapport
avec le papier utilisé, par exemple:
\setpapersize{A4}

* L'environnement changemargin décrit ci-dessous permet de
modifier localement les marges d'un document. Il prend deux
arguments, la marge gauche et la marge droite (ces arguments
peuvent prendre des valeurs négatives).

%%%% debut macro %%%%
\newenvironment{changemargin}[2]{\begin{list}{}{%
\setlength{\topsep}{0pt}%
\setlength{\leftmargin}{0pt}%
\setlength{\rightmargin}{0pt}%
\setlength{\listparindent}{\parindent}%
\setlength{\itemindent}{\parindent}%
\setlength{\parsep}{0pt plus 1pt}%
\addtolength{\leftmargin}{#1}%
\addtolength{\rightmargin}{#2}%
}\item }{\end{list}}
%%%% fin macro %%%%

Exemple :
\begin{changemargin}{2cm}{-1cm}
   Ceci permet d'augmenter la marge gauche de 2cm et de diminuer
   celle de droite de 1cm.
\end{changemargin}
A comparer avec un texte qui occupe toute la largeur de la page,
comme celui-ci.
%%%% fin exemple %%%%

* Le package 'geometry' permet de redéfinir les marges d'un
document ou de définir le layout de la page.

Exemple 1 :
\geometry{margin=5pt}
équivalent à
\geometry{hmargin=5pt, vmargin=5pt}
équivalent à
\geometry{margin={5pt,5pt}}
%%%% fin exemple %%%%

Exemple 2 :
\documentclass{report}
\usepackage{french}
\usepackage{geometry}
\geometry{scale=1.0, nohead}
\begin{document}
Ainsi, le texte apparaît beaucoup plus haut dans une page.
\end{document}
%%%% fin exemple %%%%

Exemple 3 :
\documentclass{report}
\usepackage{french}
\begin{document}
Voici une page normale pour comparer.
\end{document}
%%%% fin exemple %%%%

* Le package 'truncate' disponible sur
ftp://ftp.fdn.org/pub/CTAN/macros/latex/contrib/other/misc/ permet de fixer la
largeur d'un texte.

* Il existe également le package 'typearea' disponible sur
ftp://ftp.fdn.org/pub/CTAN/macros/latex209/contrib/script/.

* L'environnement narrow, de K. Reckdahl, ci-dessous permet
d'encapsuler des paragraphes de largeurs différentes.
%%%% debut macro %%%%
%-----------------------------------------------------------------
% \begin{narrow}{1.0in}{0.5in}   produces text which is narrowed
%          by 1.0 on left margin and 0.5 inches on right margin
% \begin{narrow}{-1.0in}{-0.5in} produces text which is widened
%          by 1.0 on left margin and 0.5 inches on right margin
% Narrow environments can be nested and are ended by \end{narrow}
%-----------------------------------------------------------------
\newenvironment{narrow}[2]{%
 \begin{list}{}{%
  \setlength{\topsep}{0pt}%
  \setlength{\leftmargin}{#1}%
  \setlength{\rightmargin}{#2}%
  \setlength{\listparindent}{\parindent}%
  \setlength{\itemindent}{\parindent}%
  \setlength{\parsep}{\parskip}%
 }%
\item[]}{\end{list}}
%%%% fin macro %%%%

# 6.8 # Comment changer l'orientation d'un document ?
-----------------------------------------------------
* Globalement, pour passer en orientation paysage, il suffit de
mettre l'option landscape dans \documentclass (LaTeX2e) ou dans
\documentstyle (LaTeX2.09).

Exemple :
\documentclass[landscape]{report}
\usepackage{french}
\begin{document}
Voici un document écrit dans un sens non conventionnel.
\end{document}
%%%% fin exemple %%%%

* Le package 'lscape' de D. Carlisle (LaTeX2e), disponible
sur ftp://ftp.fdn.org/pub/CTAN/macros/latex/packages/graphics/, permet de changer
localement d'orientation portrait vers paysage et vice versa. Il
définit l'environnement landscape.

Exemple :
\documentclass[11pt]{report}
\usepackage{lscape}

\begin{document}

\begin{landscape}
   Un petit tour à la campagne,~\ldots
\end{landscape}

et nous voici de retour dans la galerie, après un changement de
page bien évidemment.

\end{document}
%%%% fin exemple %%%%

* Il existe également le package 'rotating' disponible sur
ftp://ftp.fdn.org/pub/CTAN/macros/latex/contrib/supported/rotating/.

Exemple (tiré de la doc) :
\newcount\wang
\newsavebox{\wangtext}
\newdimen\wangspace
\def\wheel#1{\savebox{\wangtext}{#1}%
\wangspace\wd\wangtext
\advance\wangspace by 1cm%
\centerline{%
\rule{0pt}{\wangspace}%
\rule[-\wangspace]{0pt}{\wangspace}%
\wang=-180\loop\ifnum\wang<180
\rlap{\begin{rotate}{\the\wang}%
\rule{1cm}{0pt}#1\end{rotate}}%
\advance\wang by 10 \repeat}}
\wheel{Save the whale}
%%%% fin exemple %%%%

* Il existe aussi le programme docstrip disponible sur
ftp://ftp.fdn.org/pub/CTAN/macros/latex/unpacked/docstrip.tex.

* Pour le cas où vous auriez un fichier dvi en landscape, vous
pouvez obtenir un fichier postscript propre en utilisant la
commande:

Exemple :
dvips -t a4 -t landscape -o tmp.ps toto.dvi
%%%% fin exemple %%%%

# 6.9 # Comment justifier verticalement un paragraphe ?
-------------------------------------------------------
L'environnement vcenterpage ci-dessous permet de centrer
verticalement un texte sur une page seule.

%%%% debut macro %%%%
\newenvironment{vcenterpage}
{\newpage\vspace*{\fill}}
{\vspace*{\fill}\par\pagebreak}
%%%% fin macro %%%%

Exemple :
\begin{vcenterpage}
   Texte qui apparait au milieu de la page.
\end{vcenterpage}
%%%% fin exemple %%%%

# 6.10 # Comment modifier la commande \caption ?
------------------------------------------------
* Le package 'hangcaption', disponible sur
ftp://ftp.fdn.org/pub/CTAN/macros/latex209/contrib/misc/, dans lequel la commande
\isucaption remplace la commande \caption, permet de modifier la
mise en page de cette dernière. En particulier, il permet de
définir \captionwidth.

* Il existe également les packages 'caption' et 'caption2'
disponibles sur
ftp://ftp.fdn.org/pub/CTAN/macros/latex/contrib/supported/caption/ qui permettent de
modifier la commande \caption classique (largeur, style,
fonte, ...).

Exemples :
\usepackage[small,hang]{caption2}
\renewcommand{\captionfont}{\it \small}
\renewcommand{\captionlabelfont}{\it \bf \small}
\renewcommand{\captionlabeldelim}{ :}
%%%% fin exemple %%%%

Remarque : il est recommandé d'utiliser 'caption2' plutôt que
	   'caption'.

* Le package 'topcapt' disponible sur
ftp://ftp.fdn.org/pub/CTAN/macros/latex/contrib/misc/ définit la commande \topcaption qui
permet de placer le texte de la commande caption au dessus du
flottant auquel elle est rattachée.

* Le package 'french' de B. Gaulle propose la commande
\unnumberedcaptions qui permet de supprimer la numérotation des
flottants.

* Pour avoir une présentation du type :
Figure 1. Voici le texte de la légende d'une figure. Mais
          lorsque la légende est longue, elle se présente
          comme ceci.

au lieu de :
Figure 1. Voici le texte de la légende d'une figure. Mais
lorsque la légende est longue, elle se présente comme
ceci.

on peut utiliser \caption[texte]{\protect\parbox[t]{wd}{texte}}
où wd est la largeur de la boîte (cette largeur peut être adaptée
à la taille de la figure automatiquement si la figure est
elle-même dans une boite dont on récupère la largeur). \protect
est absolument nécessaire pour éviter une erreur. Le texte au
début [texte] est celui qui apparaîtra dans la table des figures
(il peut éventuellement être plus court).

Plus simplement, le package 'caption2' permet de faire la même
chose avec l'option hang.

* La variable \figurename permet de redéfinir le nom du titre de
la figure. De même pour \tablename et les tableaux.

Exemple :
\def\figurename{Croquis}
%%%% fin exemple %%%%

# 6.11 # Comment modifier un changement de page ?
-------------------------------------------------
* \newpage impose un changement brutal de page.

* \pagebreak[n] où n représente l'autorité avec laquelle on veut
changer de page (n compris entre 1 et 5). Cette commande garde
la justification, 1 impose une contrainte faible, 5 impose une
contrainte sévère.

* \nopagebreak[n] idem pour empêcher une coupure de page.

* \clearpage agit comme un \newpage mais libère le buffer de
flottants.

* \cleardoublepage fait la même chose et force un redémmarrage
sur une page impaire.

* \enlargethispage{lgr} impose à LaTeX2e de comprimer (lgr
négatif) ou d'expanser (lgr positif) le contenu d'une page.
Ceci pour éviter que la page suivante contienne trop peu de
texte.

Exemples :
\enlargethispage{1cm}	% ajoute un cm
\enlargethispage{-2\baselineskip} % supprime deux lignes dans la
				  % page
%%%% fin exemple %%%%

Remarque : \enlargethispage*{lgr} donne une autorité plus grande
	   à la commande \enlargethispage en l'obligeant à agir
	   sur les espacements élastiques verticaux contenus
	   dans la page courante.

* J.P. Drucbert donne ci-dessous un petit papier décrivant un
style block.sty bricolé à partir de macro de diverses origines
(dont D. Arseneau, assez connu). Si vous passez LaTeX sur ce
papier, block.sty sera créé. Les macros les plus intéressantes
sont \need{dim} et \lneed{N}, qui provoquent un saut de page
s'il reste moins que la dimension dim (ou N fois \baselinskip)
verticalement sur la page. Ce n'est pas parfait, mais cela m'a
rendu des services.

%%%% debut macro %%%%
\begin{filecontents}{block.sty}
\ifx\endBlock\undefined
\def\block{\begingroup%
\def\endblock{\egroup\endgroup}%
\vbox\bgroup}%
\long\def\Block{\begingroup%
\def\endBlock{\unskip\egroup\endgroup}%
\pagebreak[2]\vspace*{\parskip}\vbox\bgroup%
\par\noindent\ignorespaces}
\long\def\IBlock{\begingroup%
\def\endIBlock{\unskip\egroup\endgroup}%
\pagebreak[2]\vspace*{\parskip}\vbox\bgroup\par\ignorespaces}
\def\need#1{\ifhmode\unskip\par\fi \penalty-100 \begingroup
% preserve \dimen@, \dimen@i
   \ifdim\prevdepth>\maxdepth \dimen@i\maxdepth
      \else \dimen@i\prevdepth\fi
   \kern-\dimen@i
   \dimen@\pagegoal \advance\dimen@-\pagetotal % space left
   \ifdim #1>\dimen@
        \vfill\eject\typeout{WARNING- EJECT BY NEED}
   \fi
   \kern\dimen@i
   \endgroup}
\def\lneed#1{\need{#1\baselineskip}}
% \begin{block} ... \end{block} delimite un bloc qui restera,
%                               si possible, sur une seule page.
\long\def\TBlock{\begingroup%
\def\endTBlock{\unskip\egroup\endgroup}%
\pagebreak[2]\vspace*{\parskip}\vtop\bgroup%
\par\noindent\ignorespaces}
\else
\typeout{block.sty already loaded}
\fi
\endinput

\def\need#1{\par \penalty-100 \begingroup
% preserve \dimen@, \dimen@i
   \ifdim\prevdepth>\maxdepth \dimen@i\maxdepth
      \else \dimen@i\prevdepth\fi
   \kern-\dimen@i
   \dimen@\pagegoal \advance\dimen@-\pagetotal % space left
   \ifdim #1>\dimen@ \vfil \eject \fi
   \kern\dimen@i
   \endgroup}
\end{filecontents}

\documentclass[12pt,a4paper]{article}
\def\bs{\texttt{\char'134}}
\parskip=12pt plus1pt minus0.5pt
\usepackage[english,francais]{babel}
\usepackage[T1]{fontenc}
\usepackage[isolatin]{inputenc}
\begin{document}
\clearpage
\section{Paquetage \texttt{block}}\label{BLOCK+}
Ce paquetage offre divers outils permettant d'empêcher qu'une
rupture de page se produise à un certain endroit.

\subsection{Blocs Protégés}

Vous pouvez protéger une zone contre la rupture de page. Il
suffit pour cela de la placer dans un environnement
\texttt{Block} ou dans un environnement \texttt{IBlock}. Dans le
premier cas (\texttt{Block}), le premier paragraphe se trouvant
dans la zone ne sera pas indenté, mais le sera dans le second
cas (\texttt{IBlock}). Ceci est utile en particulier pour
éviter de séparer un texte et un exemple qui l'accompagne. Les blocs ainsi
protégés doivent, bien entendu, rester assez petits. La syntaxe
est (vous pouvez remplacer \texttt{Block} par \texttt{IBlock}):
\begin{quote}\tt
%\begin{tabular}{l}
\bs begin\{Block\}\\
\qquad\ldots\\
zone protégée\\
\qquad\ldots\\
\bs end\{Block\}
%\end{tabular}
\end{quote}

Cette méthode, très simple, a l'inconvénient de ne pas pouvoir
s'appliquer lorsque la zone à protéger doit contenir une
commande de sectionnement (c'est-à-dire du même type que
\verb|\section|), une note en bas de page, une note marginale ou
un corps mobile (figure ou table).  Dans ce cas, il faudra
utiliser une des commandes du paragraphe suivant.

\subsection{Réservations Verticales}
Vous pouvez aussi demander de changer de page (ou de colonne, si votre
document est sur deux colonnes) s'il ne reste pas verticalement assez
de place sur la page. Deux commandes de réservation verticale sont
disponibles: \verb|\need{|{\em dimension\/}\verb|}|, dont le paramètre est une
longueur, et \verb|\lneed{|{\em nombre\/}\verb|}|, dont le paramètre est
le nombre de lignes équivalant à l'espace vertical demandé (avec cette forme
le paramètre est plus facile à estimer).

\end{document}
%%%% fin macro %%%%

* Pour éviter les coupures de page (ou les coupures de colonnes
avec multicol.sty) à l'intérieur des \item dans une description
(et d'ailleurs dans toute liste), solution simple sortie du
TeXbook:
\begin{description}\interlinepenalty 10000

# 6.12 # Comment obtenir des colonnes parallèles ?
--------------------------------------------------
Le package 'parallel' disponible sur
ftp://ftp.fdn.org/pub/CTAN/macros/latex/contrib/supported/parallel/ permet d'obtenir
deux colonnes dont l'une peut contenir la traduction de l'autre.

# 6.13 # Comment définir l'espace de début de paragraphe ?
----------------------------------------------------------
* Il faut valoriser la variable \parindent.

Exemple :
\setlength{\parindent}{1cm}
%%%% fin exemple %%%%

* Pour agir localement, on peut utiliser \hspace*{lgr}.

* Le package 'indentfirst' permet de forcer LaTeX à indenter le
premier paragraphe après une nouvelle section (indentation
normalement non utilisée en typographie française).

# 6.14 # Comment supprimer la date sur une page de titre ?
----------------------------------------------------------
Il faut ajouter la commande \date{} dans le préambule du
document.

# 6.15 # Comment mettre en valeur la première lettre d'un chapitre ?
--------------------------------------------------------------------
* Il faut utiliser le package 'dropcaps' de F. Lauwers.
Il est disponible sur ftp://ftp.fdn.org/pub/CTAN/macros/latex209/contrib/dropcaps/.
Ce package est utilisable avec LaTeX2.09 et LaTeX2e.

* Les packages 'initial' et 'initials' pour LaTeX2e sont
disponibles sur CTAN. Le second que l'on peut trouver dans
ftp://ftp.fdn.org/pub/CTAN/fonts/gothic/yinit/ fait appel à des fontes yinit particulières.

* Il existe également un package 'drop' pour LaTeX2.09, mais
compatible LaTeX2e, disponible sur
ftp://ftp.fdn.org/pub/CTAN/macros/latex209/contrib/misc/.

Exemple :
\documentclass[12pt,a4paper]{article}
\usepackage{drop}
\font\largefont=yinitas % fontes yinit
begin{document}
\drop{D}OES THERE EXIST a field with 4 elements?
\end{document}
%%%% fin exemple %%%%

* 'bigdrop' accessible sur CTAN dans/CTAN/digests/ttn est une
macro TeX compatible LaTeX.

* Le package 'bigstart' pour LaTeX2.09 et LaTeX2e permet
également de faire cela.

* Il existe également le package 'picinpar' pour LaTeX2.09
disponible sur ftp://ftp.fdn.org/pub/CTAN/macros/latex209/contrib/picinpar/ et
ftp://ftp.fdn.org/pub/CTAN/systems/msdos/4alltex/disk04/.

* On peut également définir sa propre macro :
%%%% debut macro %%%%
\font\capfont=cmbx12 at 24.87 pt % or yinit, or...?
\newbox\capbox \newcount\capl \def\a{A}
\def\docappar{\medbreak\noindent\setbox\capbox\hbox{%
\capfont\a\hskip0.15em}\hangindent=\wd\capbox%
\capl=\ht\capbox\divide\capl by\baselineskip\advance\capl by1%
\hangafter=-\capl%
\hbox{\vbox to8pt{\hbox to0pt{\hss\box\capbox}\vss}}}
\def\cappar{\afterassignment\docappar\noexpand\let\a }
%%%% fin macro %%%%

Exemple :
\cappar Il était une fois un petit chaperon rouge qui avait une
grand-mère qui habitait de l'autre côté de la foret. Un jour,
alors que sa grand-mère était malade, le petit chaperon rouge
décida de lui rendre visite~\ldots
%%%% fin exemple %%%%

* Un petit dernier, le package 'dropping' qui étend le
package 'dropcaps' est disponible sur
ftp://ftp.fdn.org/pub/CTAN/macros/latex/contrib/other/dropping/.

Exemple :
\dropping{3}{\itshape{} Voici} un exemple de ce que permet de
faire le magnifique package dropping de M. Dahlgren. La commande
\texttt{\\dropping} peut prendre en argument un mot comme c'est le cas ici
ou une simple lettre.
%%%% fin exemple %%%%

* Le package 'french' de B. Gaulle propose également la commande
\lettrine.

Exemple :
\lettrine{UN jour,} mon prince~\ldots
%%%% fin exemple %%%%

* Le paquetage 'lettrine' de Daniel Flipo propose lui aussi la commande
\lettrine, mais avec une diversité d'options inconnues à french (nombre de
lignes, forme, etc.) Tiré de la doc:

Exemple :
\lettrine[lines=4, slope=-0.5em, lhang=0.5, nindent=0pt]{V}{oici}
%%%% fin exemple %%%%

* Le paquetage 'lettrine' de Daniel Flipo propose lui aussi la commande
\lettrine, mais avec une diversité d'options inconnues à french (nombre de
lignes, forme, etc.) Tiré de la doc:

Exemple :
\lettrine[lines=4, slope=-0.5em, lhang=0.5, nindent=0pt]{V}{oici}
%%%% fin exemple %%%%

# 6.16 # Comment préserver les tabulations en mode verbatim ?
-------------------------------------------------------------
* Le package 'moreverb' est disponible sur
ftp://ftp.fdn.org/pub/CTAN/macros/latex/contrib/other/misc/. Il propose notamment un
environnement verbatimtab qui permet de conserver des
tabulations.

Exemple :
\begin{verbatimtab}
int pattern(char *p, int n, int m)
{
  int orig = current_position();
  int new_pos;

  goto_line(n);

  if(p && forward_search(p) && (current_line()<m)){
    new_pos = current_position();
    goto_char(orig);
    return(new_pos);
  }
  return(-1);
}
\end{verbatimtab}
%%%% fin exemple %%%%

* Il existe également le package 'verbasef' (verbatim automatic
segmentation of external files) disponible sur CTAN. Il utilise
l'environnement figure.

* On peut également inclure les lignes suivantes dans le
préambule du document :
%%%% debut macro %%%%
\makeatletter
{\catcode`\^^I=\active
\gdef\verbatim{\catcode`\^^I=\active\def^^I{\hspace*{4em}}%
\@verbatim \frenchspacing\@vobeyspaces \@xverbatim}}
\makeatother
%%%% fin macro %%%%

# 6.17 # Comment modifier les entêtes de chapitre ?
---------------------------------------------------
Il faut modifier la macro makechapterhead.

Ajouter, par exemple, dans le preambule :
%%%% debut macro %%%%
\makeatletter
\def\@makechapterhead#1{%
  \vspace*{50\p@}%
  {\parindent \z@ \raggedright \normalfont
    \interlinepenalty\@M
    \ifnum \c@secnumdepth >\m@ne
        \Huge\bfseries \thechapter\quad
    \fi
    \Huge \bfseries #1\par\nobreak
    \vskip 40\p@
  }}

\def\@makeschapterhead#1{%
  \vspace*{50\p@}%
  {\parindent \z@ \raggedright
    \normalfont
    \interlinepenalty\@M
    \Huge \bfseries  #1\par\nobreak
    \vskip 40\p@
  }}
\makeatother
%%%% fin macro %%%%

# 6.18 # Comment définir des tabulations ?
------------------------------------------
Il faut utiliser l'environnement tabbing qui permet de placer
des marques d'alignement dans un texte.

Exemple :
\begin{tabbing}
   Voici \= des \= marques \= de tabulation \\
   \> la je m'aligne sur la première \\
   \> \> \> la sur la troisième \\
   \hspace{3cm} \= \hspace{2cm} \= \kill
   un \> autre \> exemple.
\end{tabbing}
%%%% fin exemple %%%%

# 6.19 # Comment obtenir des lettres accentuées dans tabbing ?
--------------------------------------------------------------
* Pour produire un \'{e} dans un environnement tabbing où la
commande \' a été redéfinie, il faut utiliser \a'{e} ou \a'e

* Le style suivant de J.P. Drucbert permet de remplacer
l'environnement standard tabbing par l'environnement Tabbing
dans lequel les commandes \` \' \> \< \= \+ \- sont remplacées
par \TAB` \TAB' \TAB> \TAB< \TAB= \TAB+ \TAB- ce qui permet de
préserver les commandes d'accent (\' \` ou \=).

%%%% debut macro %%%%
\ProvidesPackage{Tabbing}[1996/01/16]
\NeedsTeXFormat{LaTeX2e}[1995/12/01]
\gdef\Tabbing{\lineskip \z@skip
%     \let\>\@rtab
%     \let\<\@ltab
%     \let\=\@settab
%     \let\+\@tabplus
%     \let\-\@tabminus
%     \let\`\@tabrj
%     \let\'\@tablab
\def\TAB##1{\ifx ##1>\@rtab\else
            \ifx ##1<\@ltab\else
            \ifx ##1=\@settab\else
            \ifx ##1+\@tabplus\else
            \ifx ##1-\@tabminus\else
            \ifx ##1`\@tabrj\else
            \ifx ##1'\@tablab\else
                         \PackageError{Tabbing}%
                         {Bad argument ##1 for Tabbing
			 specification} \fi\fi\fi\fi\fi\fi\fi}
     \let\\=\@tabcr
     \global\@hightab\@firsttab
     \global\@nxttabmar\@firsttab
     \dimen\@firsttab\@totalleftmargin
     \global\@tabpush\z@ \global\@rjfieldfalse
     \trivlist \item\relax
     \if@minipage\else\vskip\parskip\fi
     \setbox\@tabfbox\hbox{\rlap{\indent\hskip\@totalleftmargin
       \the\everypar}}\def\@itemfudge{\box\@tabfbox}%
     \@startline\ignorespaces}
\gdef\endTabbing{%
  \@stopline\ifnum\@tabpush >\z@ \@badpoptabs \fi\endtrivlist}
\endinput
%%%% fin macro %%%%

# 6.20 # Comment encadrer du texte ?
------------------------------------
* Une solution consiste à définir un tableau d'une seule
cellule.

* On peut aussi utiliser :
\fbox{
\begin{minipage}{0.7\textwidth}
   Texte...
\end{minipage}
}

* On peut également se définir son propre environnement.

Exemple :
\newsavebox{\fmbox}
\newenvironment{fmpage}[1]
     {\begin{lrbox}{\fmbox}\begin{minipage}{#1}}
     {\end{minipage}\end{lrbox}\fbox{\usebox{\fmbox}}}

Utilisation :
\begin{fmpage}{3cm}
   Texte à encadrer dans une boîte ne dépassant pas 3
   centimètres de large.
\end{fmpage}
%%%% fin exemple %%%%

* Il existe également le package 'fancybox', disponible sur
ftp://ftp.fdn.org/pub/CTAN/macros/latex/contrib/others/seminar/inputs/, qui définit
des fonctions telles que \shadowbox, \doublebox, \ovalbox,...

Exemple :
\shadowbox{Texte ombré.}
\doublebox{Texte doublement encadré.}
\ovalbox{Texte dans un cadre aux coins arrondis.}
%%%% fin exemple %%%%

* Le package 'boxedminipage' est un vieux style LaTeX2.09
disponible sur ftp://ftp.fdn.org/pub/CTAN/macros/latex209/contrib/misc/.

* Voir également le package 'hh' disponible sur
ftp://ftp.fdn.org/pub/CTAN/macros/latex/contrib/supported/hh/.

* Le package 'niceframe' disponible sur
ftp://ftp.fdn.org/pub/CTAN/macros/latex/contrib/supported/niceframe/ permet de
définir des cadres pleine page.

* Pour encadrer un texte pouvant s'étendre sur plusieurs pages,
il existe le package 'eclbkbox' :
%%%% debut macro %%%%
% eclbkbox.sty by Hideki Isozaki, 1992
% Date: May  28, 1993

\newbox\bk@bxb
\newbox\bk@bxa
\newif\if@bkcont
\newif\ifbkcount
\newcount\bk@lcnt

\def\breakboxskip{2pt}
\def\breakboxparindent{1.8em}

\def\breakbox{\vskip\breakboxskip\relax
\setbox\bk@bxb\vbox\bgroup
\advance\linewidth -2\fboxrule
\advance\linewidth -2\fboxsep
\hsize\linewidth\@parboxrestore
\parindent\breakboxparindent\relax}

% \@tempdimb: amount of vertical skip
% between the first line (\bk@bxa) and the rest (\bk@bxb)
\def\bk@split{%
\@tempdimb\ht\bk@bxb % height of original box
\advance\@tempdimb\dp\bk@bxb
\setbox\bk@bxa\vsplit\bk@bxb to\z@ % split it
\setbox\bk@bxa\vbox{\unvbox\bk@bxa}% recover height & depth of \bk@bxa
\setbox\@tempboxa\vbox{\copy\bk@bxa\copy\bk@bxb}% naive concatenation
\advance\@tempdimb-\ht\@tempboxa
\advance\@tempdimb-\dp\@tempboxa}% gap between two boxes

% \@tempdima: height of the first line (\bk@bxa) + fboxsep
\def\bk@addfsepht{%
     \setbox\bk@bxa\vbox{\vskip\fboxsep\box\bk@bxa}}

\def\bk@addskipht{%
     \setbox\bk@bxa\vbox{\vskip\@tempdimb\box\bk@bxa}}

% \@tempdima: depth of the first line (\bk@bxa) + fboxsep
\def\bk@addfsepdp{%
     \@tempdima\dp\bk@bxa
     \advance\@tempdima\fboxsep
     \dp\bk@bxa\@tempdima}

% \@tempdima: depth of the first line (\bk@bxa) + vertical skip
\def\bk@addskipdp{%
     \@tempdima\dp\bk@bxa
     \advance\@tempdima\@tempdimb
     \dp\bk@bxa\@tempdima}

\def\bk@line{%
    \hbox to \linewidth{\ifbkcount\smash{\llap{\the\bk@lcnt\ }}\fi
    \vrule \@width\fboxrule\hskip\fboxsep
    \box\bk@bxa\hfil
    \hskip\fboxsep\vrule \@width\fboxrule}}

\def\endbreakbox{\egroup
\ifhmode\par\fi{\noindent\bk@lcnt\@ne
\@bkconttrue\baselineskip\z@\lineskiplimit\z@
\lineskip\z@\vfuzz\maxdimen
\bk@split\bk@addfsepht\bk@addskipdp
\ifvoid\bk@bxb      % Only one line
\def\bk@fstln{\bk@addfsepdp
\vbox{\hrule\@height\fboxrule\bk@line\hrule\@height\fboxrule}}%
\else               % More than one line
\def\bk@fstln{\vbox{\hrule\@height\fboxrule\bk@line}\hfil
\advance\bk@lcnt\@ne
\loop
 \bk@split\bk@addskipdp\leavevmode
\ifvoid\bk@bxb      % The last line
 \@bkcontfalse\bk@addfsepdp
 \vtop{\bk@line\hrule\@height\fboxrule}%
\else               % 2,...,(n-1)
 \bk@line
\fi
 \hfil\advance\bk@lcnt\@ne
\if@bkcont\repeat}%
\fi
\leavevmode\bk@fstln\par}\vskip\breakboxskip\relax}

\bkcountfalse
%%%% fin macro %%%%

L'encadrement du texte est obtenu par l'environnement breakbox.

\bkcounttrue : les lignes sont numérotées.
\bkcountfalse : elles ne le sont pas (défaut).

On peut emboîter des environnements breakbox.

# 6.21 # Comment gérer des URL WWW ?
------------------------------------
* Il existe le package 'url' disponible sur
ftp://ftp.fdn.org/pub/CTAN/macros/latex/contrib/other/misc/ qui permet de gérer les
coupures des URL WWW un peu longues. Il est également capable de
gérer les adresses e-mail, les liens hypertexte, les noms de
directories. En outre, cet outil est paramètrable.

Exemple :
\usepackage{url}
\urlstyle{sf}
..
\url{http://hostname/~username}
%%%% fin exemple %%%%

* Le package 'path' disponible sur ftp://ftp.fdn.org/pub/CTAN/macros/eplain/
offre les mêmes possibilités.

* La macro \discretionary permet également de dire comment
couper une chaîne de caractères :
\discretionary{Avant la coupure}{après}{s'il n'y en a pas}

Remarque : \- est défini comme \discretionary {-}{}{}

# 6.22 # Comment mettre en page des exercices dont les solutions sont reportées dans un autre paragraphe ?
----------------------------------------------------------------------------------------------------------
L'objectif ici est de pouvoir saisir dans le fichier source les
textes des exercices suivis de leurs solutions, alors qu'au
niveau de la mise en page du document, les solutions
apparaîssent groupées dans un autre paragraphe/chapitre.

* Le package 'answers', de M. Piff, disponible sur
ftp://ftp.fdn.org/pub/CTAN/macros/latex/contrib/supported/answers/ permet également de
réaliser ce genre d'exercice. Il permet entre autres :
+ d'avoir plusieurs types de solutions (ex: réponse numérique
seule ou détail),
+ d'inclure les solutions (l'une, l'autre ou les deux dans
le cas précité) dans le texte (après une marque spécifique
si désiré),
+ mettre les solutions à la fin,
+ ne pas mettre les solutions.

Exemple :
%%
%% This is file `ansexam2.tex',
%% generated with the docstrip utility.
%%
%% The original source files were:
%%
%% answers.dtx  (with options: `ex2')
%%
\documentclass[12pt,a4paper]{article}
\usepackage{answers}%\usepackage[nosolutionfiles]{answers}
% def d'un environnement Exercise numerote
\newtheorem{Exc}{Exercise}
\newenvironment{Ex}{\begin{Exc}\normalfont}{\end{Exc}}
% Trois types de solutions sont proposes
\Newassociation{solution}{Soln}{test}
\Newassociation{hint}{Hint}{test}
\Newassociation{Solution}{sSol}{testtwo}
\newcommand{\prehint}{~[Hint]}
\newcommand{\presolution}{~[Solution]}
\newcommand{\preSolution}{~[Homework]}
% test
\newcommand{\Opentesthook}[2]%
   {\Writetofile{#1}{\protect\section{#1: #2}}}
% introduction de la solution
\renewcommand{\Solnlabel}[1]{\emph{Solution #1}}
\renewcommand{\Hintlabel}[1]{\emph{Hint #1}}
\renewcommand{\sSollabel}[1]{\emph{Solution to #1}}

\begin{document}
% gestion des fichiers contenant les solutions
   \Opensolutionfile{test}[ans2]{Solutions}
   \Writetofile{test}{\protect\subsection{Some Solutions}}
   \Opensolutionfile{testtwo}[ans2x]
   \Writetofile{testtwo}{%
      \protect\subsection{Extra Solutions}}

      % Exercices
   \section{Exercises}
   \begin{Ex}
      An exercise with a solution.
      \begin{solution}
         This is a solution.
         \relax{}
      \end{solution}
   \end{Ex}
   \begin{Ex}
      An exercise with a hint and a secret solution.
      \begin{hint}
         This is a hint.
      \end{hint}
      \begin{Solution}
         This is a secret solution.
      \end{Solution}
   \end{Ex}
   \begin{Ex}
      An exercise with a hint.
      \begin{hint}
         This is a hint.
      \end{hint}
   \end{Ex}
   % gestion des fichiers contenant les solutions
   \Closesolutionfile{test}
   \Readsolutionfile{test}
   % \clearpage
   \hrule
   \Closesolutionfile{testtwo}
   \Readsolutionfile{testtwo}
\end{document}
%%
%% End of file `ansexam2.tex'.

%%%% fin exemple %%%%

* On peut également trouver le package 'exam' sur
ftp://ftp.fdn.org/pub/CTAN/macros/latex/contrib/supported/exams/.

# 6.23 # Comment positionner un objet dans une page ?
-----------------------------------------------------
* Pour pouvoir positionner un objet aux coordonnées x,y par
rapport au coin supérieur gauche d'une page, il suffit d'utiliser
le package 'atxy' disponible par ftp sur
ftp://ftp.univ-orleans.fr/pub/tex/PC/AsTeX/Paq_Base/ dans
le fichier l209misc.zip.

Exemple :
\atxy(3cm,2.5cm) {toto adresse toto}
\atxy(3cm,4cm) {date}
%%%% fin exemple %%%%

Remarque : petit défaut, si le document ne contient que des
	   commandes \atxy il n'y a pas de dvi généré. Il faut
	   donc ajouter n'importe quoi au début du document, ~
	   par exemple, pour que ça marche.

# 6.24 # Comment lier le placement des flottants aux sections ?
---------------------------------------------------------------
Pour lier la position des éléments flottants aux limites de
sections, D. Arseneau a développé le package 'placeins'
disponible sur ftp://ftp.fdn.org/pub/CTAN/macros/latex/contrib/other/misc/. Ce
package définit la commande \FloatBarrier qui force le placement
des flottants avant son appel.

# 6.25 # Comment griser le fond (background) d'un paragraphe ?
--------------------------------------------------------------
* Le package 'psboxit', disponible sur
ftp://ftp.fdn.org/pub/CTAN/macros/latex2.09/contrib/misc/ et en particulier
l'environnement "boxitpara" permet de faire cela.

* Le package 'shadbox' disponible sur CTAN dans
ftp://ftp.fdn.org/pub/CTAN/macros/latex/contrib/supported/shadbox/ permet de griser toute
boîte, texte, figure, ....

* Le package 'shading' disponible sur
ftp://ftp.fdn.org/pub/CTAN/macros/latex209/contrib/shading/ permet de griser un
paragraphe.

* De même, le package 'shade', de P. Schmitt, est disponible sur
ftp://ftp.fdn.org/pub/CTAN/macros/generic/.

* On peut également utiliser le package 'color'.

Exemple :
\colorbox[gray]{0.5}{some words}
%%%% fin exemple %%%%

# 6.26 # Comment modifier l'espace inter-colonnes ?
---------------------------------------------------
Il faut modifier la variable \columnsep.

Exemple :
\addtolength{\columnsep}{5mm}
%%%% fin exemple %%%%

# 6.27 # Comment modifier les environnements de liste ?
-------------------------------------------------------
* L'environnement list permet de définir son propre style de
liste. Sa syntaxe est la suivante :
\begin{list}{label}{mep}\end{list}
+ l'argument label permet de définir le symbole qui sera
associé à chaque élément de la liste.
+ mep permet de définir la mise en page des éléments de la
liste. Les paramètres utilisés pour définir cette mise en page
sont les suivants :
\topsep espace vertical supplémentaire (ajoute à \parskip)
	inséré entre le texte précédant la liste et le 1er objet
	de la liste
\partosep espace vertical supplémentaire inséré devant la liste
	si celle-ci est précédée d'une ligne blanche
\itemsep espace vertical supplémentaire (ajouté à \parsep)
	inséré entre les éléments d'une liste.

Exemple :
\newenvironment{maliste}%
{ \begin{list}%
	{$\bullet$}%
	{\setlength{\labelwidth}{30pt}%
	 \setlength{\leftmargin}{35pt}%
	 \setlength{\itemsep}{\parsep}}}%
{ \end{list} }

Utilisation :
\begin{maliste}
   \item premier élément
   \item deuxième élément
   \begin{maliste}
      \item petit 1
      \item petit 2
   \end{maliste}
\end{maliste}
%%%% fin exemple %%%%

* Le petit bout de code ci-dessous, de M. Boyer
(mboyer@robot.ireq.ca), définit les commandes :
+ \noitemsep pour supprimer tout espacement vertical entre les
items des environnements \itemize, \enumerate et \description.
+ \doitemsep pour les remettre.
Pour l'utiliser, il suffit de le sauvegarder dans un fichier
.sty et de l'inclure dans son document par une commande
\usepackage.

%%%% debut macro %%%%
%% ----------------------------------------------------
%% Copyright (c) 1993 Hydro-Quebec mboyer@robot.ireq.ca
%% ----------------------------------------------------

%% Bring items closer together in list environments
% Prevent infinite loops
\let\orig@Itemize =\itemize
\let\orig@Enumerate =\enumerate
\let\orig@Description =\description
% Zero the vertical spacing parameters
\def\Nospacing{\itemsep=0pt\topsep=0pt\partopsep=0pt%
\parskip=0pt\parsep=0pt}
% Redefinition de art12.sty pour commencer a la marge de gauche
%\leftmargini 1.2em      % 2.5em

\def\noitemsep{
% Redefine the environments in terms of the original values
\renewenvironment{itemize}{\orig@Itemize\Nospacing}{\endlist}
\renewenvironment{enumerate}{\orig@Enumerate\Nospacing}{\endlist}
\renewenvironment{description}{\orig@Description\Nospacing}%
{\endlist}
}

\def\doitemsep{
% Redefine the environments to the original values
\renewenvironment{itemize}{\orig@Itemize}{\endlist}
\renewenvironment{enumerate}{\orig@Enumerate}{\endlist}
\renewenvironment{description}{\orig@Description}{\endlist}
}
%%%% fin macro %%%%

* La macro ci-dessous de T. Murphy permet de remplacer les
numéros de l'environnement enumerate par des caractères grecs :
%%%% debut macro %%%%
\makeatletter
\def\greek#1{\expandafter\@greek\csname c@#1\endcsname}
\def\@greek#1{\ifcase#1\or$\alpha$\or$\beta$\fi}% as many as you
% need
\renewcommand{\theenumi}{\greek{enumi}}
\makeatother
%%%% fin macro %%%%

* Les définitions suivantes :
\renewcommand{\labelitemi}{\textbullet}
\renewcommand{\labelitemii}{---}
\renewcommand{\labelitemiii}{votre-label-pour-le-niveau-iii}
\renewcommand{\labelitemiv}{votre-label-pour-le-niveau-iv}
permettent de redéfinir les caractères utilisés par
l'environnement itemize pour ces différents niveaux
d'encapsulation.

Remarque : avec certains packages comme french, il faut placer
	   ces nouvelles définitions de commande après le
	   \begin{document}.

De même, utilisez \descriptionlabel pour changer le style des
étiquettes de l'environment description.

Exemple :
\renewcommand\descriptionlabel[1]{\hspace\labelsep\normalfont%
\itshape #1:}
produit des étiquettes en italique, avec deux points :
\begin{description}
   \item[Carte maîtresse] As
   \item[Carte maîtresse à l'atout] Valet
\end{description}
%%%% fin exemple %%%%

* Pour réduire globalement l'espace entre les items d'une liste
on peut également utiliser le bout de code suivant de M. Wooding
%%%% debut macro %%%%
\makeatletter
\toks@\expandafter{\@listI}
\edef\@listI{\the\toks@\setlength{\parsep}{1pt}}
\makeatother
%%%% fin macro %%%%

* Le package 'mdwlist', de M. Wooding, disponible sur
ftp://ftp.fdn.org/pub/CTAN/macros/latex/contrib/supported/mdwtools/ permet de
redéfinir certains paramètres de mise en page des listes qui ne
sont pas faciles d'accès sous LaTeX.

# 6.28 # Comment souligner plusieurs lignes ?
---------------------------------------------
Pour souligner un texte qui comprend des retours à la ligne, il
faut utiliser le package 'ulem' disponible sur
ftp://ftp.fdn.org/pub/CTAN/macros/latex/contrib/other/misc/. Ce package redéfinit en
fait le mode emphasize. Les commandes \normalem et \ULforem
permettent de passer du mode \emph classique au mode \emph
souligné. En mode souligné, la commande devient paramétrable pour
changer le style du souligné ou biffer des mots.

Exemple :
\documentclass{report}
\usepackage{french}
\usepackage[normalem]{ulem}
\pagestyle{empty}
\begin{document}

\normalem
Voici le mode "\emph{emphasize}" classique.

\ULforem
Voici le mode "\emph{emphasize}" souligné. \emph{Il permet
également de gérer les retours à la ligne tout en restant dans
le style emphasize.}

Les autres possibilités sont les suivantes~:
\begin{itemize}
   \item vagues~: \uwave{texte}
   \item barré~: \sout{texte}
   \item rayé~: \xout{texte}
\end{itemize}

\end{document}
%%%% fin exemple %%%%

Remarque : \underline ne permet pas de gérer les retours à la
	   ligne du fait qu'il encapsule le texte passé en
	   argument dans une boîte.

# 6.29 # Comment réaliser des onglets ?
---------------------------------------
* Pour insérer un carré noir, décalé vers le bas à chaque
nouveau chapitre, le long de la marge des pages de droite d'un
document, on peut utiliser le package 'fancyhdr' ou
'fancyheadings'. Ces packages sont disponibles sur
ftp://ftp.fdn.org/pub/CTAN/macros/latex/contrib/supported/fancyhdr/ et
ftp://ftp.fdn.org/pub/CTAN/macros/latex/contrib/other/.

Remarque : On note toutefois qu'il est peu probable qu'une
	   imprimante puisse accéder au ras de la marge. La
	   solution consiste alors à définir un format de
	   document plus petit et utiliser le massicot.
	   Attention dans ce cas lors de la définition des
	   marges.

* On peut également utiliser le package 'onglet' défini
ci-dessous par B. Bayart. Celui-ci nécessite le package
'everyshi' disponible sur
ftp://ftp.fdn.org/pub/CTAN/macros/latex/contrib/supported/everyshi/.

%%%% debut macro %%%%
\ProvidesPackage{onglet}[1996/07/25 B. Bayart]
\RequirePackage{everyshi}

\newcounter{maxchapter}
\newcounter{tmpa}
\newlength{\basehauteur}
\setlength{\basehauteur}{1cm}
\newlength{\ajoutdroite}
\newlength{\htcclv}
\def\concatener{%
  \setlength{\ajoutdroite}{\textheight}
  \divide\ajoutdroite by \basehauteur
  \setcounter{maxchapter}{\number\ajoutdroite}
  \setcounter{tmpa}{\value{chapter}}
  \addtocounter{tmpa}{-1}
  \divide\value{tmpa} by\value{maxchapter}
  \multiply\value{tmpa} by\value{maxchapter}
  \advance\value{tmpa} by -\value{chapter}
  \addtocounter{tmpa}{-1}
  \setlength{\ajoutdroite}{\paperwidth}
  \setlength{\htcclv}{\ht255}
  \addtolength{\ajoutdroite}{-\wd255}
  \addtolength{\ajoutdroite}{-1in}
  \addtolength{\ajoutdroite}{-1.5cm}
  \setbox255=\vbox{\hbox to \wd255{%
    \box255%\relax
    \rlap{\vbox to \htcclv{%
      \vskip-\value{tmpa}\basehauteur
      \hbox{%
        \hskip\ajoutdroite\relax
        \usebox{\laboite}%
      }%
      \vfill
    }}%
  \hfill}}%
}
\newsavebox{\laboite}
\def\faireboite{\sbox{\laboite}%
{\hbox to 1.5cm{\let\protect\relax
\huge\thechapter\hfill\vrule height 1em depth 0pt width 5mm}}}

\AtBeginDocument{\EveryShipout{\faireboite\concatener}}
\endinput
%%%% fin macro %%%%

# 6.30 # Comment réaliser des QCM ?
-----------------------------------
Il existe le package 'exam' disponible sur
ftp://ftp.fdn.org/pub/CTAN/macros/latex/contrib/supported/exams/. Ce package permet
entre autres de paramétrer les questions de manière à ce que les
propositions apparaîssent dans un ordre aléatoire.

Exemple :
Combien le cheval possède t'il de pattes ?
\begin{choice}[\random]
   \baditem{deux pattes}
   \baditem{quatres pattes}
   \baditem{zéro patte}
\end{choice}
%%%% fin exemple %%%%

# 6.31 # Comment modifier l'orientation des flottants ?
-------------------------------------------------------
* Le package 'rotating' disponible sur
ftp://ftp.fdn.org/pub/CTAN/macros/latex/contrib/supported/rotating/ offre deux
environnements sidewaysfigure pour les figures et sidewaystable
pour les tableaux. Les figures ou tableaux sont alors insérés sur
une page séparée.

Exemple :
\begin{sidewaystable}
   \begin{tabular}{|c|c|}
      \hline
      Un & Deux \\
      Trois & Quatre \\
      \hline
   \end{tabular}
\end{sidewaystable}
%%%% fin exemple %%%%

* On peut également utiliser l'environnement sideways du
package 'rotating'. De la même manière cet environnement génère
une page séparée.

Exemple :
\begin{figure}
  \begin{sideways}
     \includegraphics{foobar.ps}
  \end{sideways}
\end{figure}
%%%% fin exemple %%%%

Remarque : il est fréquent que les visualiseurs de fichiers dvi
	   ne sachent pas gérer les changements d'orientation à
	   l'intérieur d'un document. Il faut pour voir le
	   résultat définitif utiliser un visualiseur de
	   postscript.

# 6.32 # Comment faire référence plusieurs fois à la même note de bas de page ?
-------------------------------------------------------------------------------
* Il faut utiliser les commandes \footnotemark[] et
\footnotetext[]{}. \footnotemark permet de gérer le compteur de
notes et \footnotetext permet d'insérer le texte correspondant
en bas de page.

Exemple :
bla blabla\footnotemark[1] bla bla bla blablabla\footnotemark[2]
bla bla bla bla blabla\footnotemark[1] bla.

\footnotetext[1]{double bla}
\footnotetext[2]{triple bla}
%%%% fin exemple %%%%

* Avec french, on peut utiliser la commande \refmark.

Exemple :
le vrai appel\footnote{C'est une note en bas de
page\label{manote}} et le deuxieme appel\refmark{manote}
%%%% fin exemple %%%%

Remarque : Si vous n'utilisez pas french, cette macro \refmark
	   est décrite dans le cahier Gutenberg numéro 15, avril
	   1993, page 52. Ce cahier est accessible à l'URL :
	   http://www.gutenberg.eu.org/publications/.

# 6.33 # Comment éviter les orphelins en début ou fin de page ?
---------------------------------------------------------------
* Il suffit d'ajouter les lignes :
%%%% debut macro %%%%
\widowpenalty=10000
\clubpenalty=10000
\raggedbottom
%%%% fin macro %%%%
dans le préambule du document.

* Pour agir localement, on peut également utiliser la commande
\enlargethispage (cf. question 6.11).

# 6.34 # Comment définir de nouveaux flottants ?
------------------------------------------------
Le package 'float' de G. Williams, disponible sur
ftp://ftp.fdn.org/pub/CTAN/macros/latex/contrib/supported/float/, permet de définir
de nouveaux objets flottants. Il définit notamment des
environnements permettant d'encadrer des objets ou de les séparer
du reste du texte par des lignes horizontales.

Exemple :
\documentclass[11pt]{report}
\usepackage{float}
\usepackage{french}

\floatstyle{ruled} % pour que mes flottants soient séparés du
		   % texte par des lignes.
\newfloat{important}{htbp}{loi}[section]
	% important est le nom de mon nouvel environnement
	% htbp sont les options de placement de mon flottant
	% loi est l'extension du fichier qui sera utilise pour
	% construire la liste de mes flottants
	% section est le niveau duquel dependra la numerotation
	% de mes flottants
\floatname{important}{Important} % titre du caption

\begin{document}

\listof{important}{Liste des textes importants.} % titre de la
	% liste de mes flottants.

\chapter{Règles de bases.}

\section{Hommes célèbres.}

   Parmi les citations des hommes célèbres dans le domaine que
   nous étudions actuellement, il faut retenir celle de M.
   Maxime rappelée dans le cadre~\ref{imp-max}.

   \begin{important}
      M. Maxime a dit un jour~:
      \begin{quote}
	 Chacun doit se faire ses propres raisons.
      \end{quote}
      \caption{Adage de M. Maxime.}
   \end{important}

   Notez que~\ldots
\end{document}
%%%% fin exemple %%%%

# 6.35 # Comment utiliser la commande \caption hors d'un environnement flottant ?
---------------------------------------------------------------------------------
* Pour pouvoir attacher un titre de style table à un
environnement non flottant (i.e. autre que figure ou table), il
faut définir :
%%%% debut macro %%%%
\makeatletter
\def\@captype{table}
\makeatother
%%%% fin macro %%%%

Exemple :
Texte sur les couleurs.

\begin{itemize}
   \item rouge
   \item vert
   \item bleu
\end{itemize}
\caption{Liste des couleurs primaires.}

Texte.
%%%% fin exemple %%%%

* La macro suivante propose une autre possibilité pour obtenir
un titre de style table :
%%%% debut macro %%%%
\makeatletter
\def\captionof#1#2{{\def\@captype{#1}#2}}
\makeatother
%%%% fin macro %%%%

Exemple :
\begin{document}
un texte

\captionof{table}{\caption{un titre ici}\label{foo}}

un autre texte
\end{document}
%%%% fin exemple %%%%

* Le package 'capt-of' disponible sur
ftp://ftp.fdn.org/pub/CTAN/macros/latex/contrib/other/misc/ définit la commande
\captionof.

# 6.36 # Comment encadrer un objet flottant ?
---------------------------------------------
* Il faut utiliser le package 'float' qui définit le style
boxed. Ce package est disponible sur
ftp://ftp.fdn.org/pub/CTAN/macros/latex/contrib/supported/float/. Voir la question
6.34 pour un exemple plus complet.

Exemple :
Dans le préambule :
\floatstyle{ruled}
\newfloat{nom-flottant}{placement}{ext-fic}[numerotation]
dans le corps du document :
\begin{nom-flottant}

\end{nom-flottant}
%%%% fin exemple %%%%

* Si le titre accompagnant le flottant peut être à l'extérieur
du cadre désiré, on peut alors utiliser \fbox dans la
déclaration du flottant.

Exemple :
\begin{table}
   \begin{center}
      \fbox{
      \begin{tabular}{cll}
	 &1 case & 2 cases \\
	 &3 cases  & 4 cases \\\hline
	 Total~: & 4 cases & 6 cases \\
      \end{tabular}
      }
      \end{center}
   \caption{Titre extérieur.}
\end{table}
%%%% fin exemple %%%%

* M. Loreti propose également le package 'bigbox' ci-dessous :
%%%% debut macro %%%%
% I like to put boxes around all of my figures and algorithms in
% reports, and finally came up with my 'bigbox' environment.  It
% makes a box as wide as the current text and sets the stuff
% inside with a narrower width.
%
% USAGE:
%
% \begin{figure}        (or {table}, or {center}, or ...)
%   \begin{bigbox}
%     \begin{tabbing}
%       ...
%     \end{tabbing}
%   \end{bigbox}
%   \caption{XXX}
% \end{figure}
%
%%% BIGBOX - environment
%%%
%%% TOM SHEFFLER
%%% Carnegie Mellon, Aug 1990
%%%
%%% Make an environment for boxing a figure and setting it in
%%% a narrower width.
\newdimen\boxfigwidth % width of figure box

\def\bigbox{\begingroup
  % Figure out how wide to set the box in
  \boxfigwidth=\hsize
  \advance\boxfigwidth by -2\fboxrule
  \advance\boxfigwidth by -2\fboxsep
  \setbox4=\vbox\bgroup\hsize\boxfigwidth
  % Make an invisible hrule so that the box is exactly this wide
  \hrule height0pt width\boxfigwidth\smallskip%
% Some environments like TABBING and other LIST environments
% use this measure of line size - \LINEWIDTH=\HSIZE-\LEFTMARGIN-\RIGHTMARGIN?
  \linewidth=\boxfigwidth
}
\def\endbigbox{\smallskip\egroup\fbox{\box4}\endgroup}
%%%% fin macro %%%%

Exemple :
\documentclass[11pt]{report}
\usepackage{graphics}
\usepackage{bigbox}
\usepackage{french}
\pagestyle{empty}
\begin{document}

\begin{figure}
   \begin{bigbox}
      \includegraphics{../foobar.ps}
   \end{bigbox}
   \caption{Dessin.}
\end{figure}

\end{document}
%%%% fin exemple %%%%

# 6.37 # Comment changer l'orientation d'un caption ?
-----------------------------------------------------
La commande \rotcaption fournie avec le package 'rotating'
permet de changer l'orientation d'un caption.

Exemple :
\begin{figure}
   \centering
   \begin{minipage}[c]{1in}
      \includegraphics[angle=90,width=\linewidth]{coco.ps}
   \end{minipage}
   \begin{minipage}
      \rotcaption{Ma jolie figure coco}
      \label{coco}
   \end{minipage}
\end{figure}
%%%% fin exemple %%%%

# 6.38 # Comment mettre en page un programme ?
----------------------------------------------
Les packages 'program' (disponible sur
ftp://ftp.fdn.org/pub/CTAN/macros/latex/contrib/supported/program/) et 'programs'
permettent de mettre en reliefs des mots clés, d'utiliser des
mathématiques dans des algorithmes, etc.

# 6.39 # Comment inclure le source d'un programme ?
---------------------------------------------------
Les packages 'lgrind' et 'listings' répondent à la question
(disponibles sur ftp://ftp.fdn.org/pub/CTAN/support/lgrind/ et
ftp://ftp.fdn.org/pub/CTAN/macros/latex/contrib/supported/listings/).  Ils
reconnaissent tous les 2 à peu près 40 langages différents.

Le premier est composé d'un exécutable, 'lgrind' qui permet de
transformer le source en question, par exemple, monfichier.c, en
monfichier.tex, que l'on inclut directement dans son fichier
LaTeX, à l'aide d'une commande appropriée. L'inconvénient, est
qu'évidemment, il y a un fichier .tex qui est généré en plus.

Notes : - utiliser au-moins la version 3.6 ;
	- on peut paramétrer lgrind avec le fichier lgrindef. Par
	  défaut, l'auteur a jugé utile de transformer la lettre
	  « à » en $\alpha$. Il suffit donc de commenter cette
	  option à la fin de ce fichier pour éviter cela.

Le deuxième ne nécessite pas de recompilation. On inclut
directement le source, avec une commande adéquate. Avec ce
package, on peut même définir son propre langage, le faire
hériter d'un autre langage, avec des mots-clés spécifiques, etc.

À noter que, moyennant quelques paramètres à ajouter, il est tout
à fait possible de faire en sorte que ces packages reconnaissent
les fontes 8 bits. On peut lui faire comprendre par exemple, que
lorsqu'il rencontre le caractère « ¹ », il le transforme en
$\mathonesuperior$.

# 6.40 # Comment obtenir un espace insécable ?
----------------------------------------------
Le caractère ~ est interprété par LaTeX et permet d'obtenir un
espace insécable.

Exemple :
Comme le montre l'exemple suivant~: $a + 2 = 0$.
%%%% fin exemple %%%%

# 6.41 # Comment insérer une page blanche ?
-------------------------------------------
* Pour forcer LaTeX à laisser une page blanche dans un document,
il faut utiliser successivement les trois commandes :
\newpage
\strut ou ~ ou \mbox{} ou \null
\newpage

Le principe est qu'il y ait quelque chose d'invisible sur la
page blanche pour que LaTeX la prenne en compte.

# 6.42 # Comment supprimer l'indentation des paragraphes ?
----------------------------------------------------------
* \noindent au début du texte permet de ne pas indenter un
paragraphe.

* Pour systématiquement supprimer l'indentation du premier
paragraphe d'une section par exemple, il faut redéfinir la
commande \section. Il faut que le 4ème paramètre de
\@startsection soit une distance négative, pour qu'il n'y ait
pas de retrait au premier paragraphe du texte qui suit :

%%%% debut macro %%%%
\makeatletter
\renewcommand\section{\@startsection {section}{1}{\z@}%
                           {-3.5ex \@plus -1ex \@minus -.2ex}%
%%%%%%                      ^^^^ (4eme parametre)
                           {2.3ex \@plus.2ex}%
                           {\normalfont\Large\bfseries}}
\makeatother
%%%% fin macro %%%%

# 6.43 # Comment modifier l'espacement entre caractères ?
---------------------------------------------------------
* Le package 'letterspace' permet de modifier l'espacement entre
mots ou caractères.

* Le package 'trackin' disponible sur
ftp://ftp.fdn.org/pub/CTAN/macros/latex/contrib/other/tracking/ permet de jouer sur
les espacements dans les mots ou les phrases pour les ajuster
dans une longueur spécifiée.

# 6.44 # Comment supprimer certaines coupures de mots ?
-------------------------------------------------------
* Pour agir sur un mot particulier, il faut utiliser la commande
\hyphenation.

Exemple :
Visualisation des coupures :
+\showhyphens{mousehole AlGaAs GaAs GaInP AlInP}

  Underfull \hbox (badness 10000) detected at line 0
  [] \OT1/cmr/m/n/10 mouse-hole Al-GaAs GaAs GaInP Al-InP

Interdiction de certaines coupures :
+\hyphenation{AlGaAs GaAs GaInP AlInP}
+\showhyphens{mousehole AlGaAs GaAs GaInP AlInP}

  Underfull \hbox (badness 10000) detected at line 0
  [] \OT1/cmr/m/n/10 mouse-hole AlGaAs GaAs GaInP AlInP
%%%% fin exemple %%%%

* Moins propre mais tout aussi efficace, on peut inclure le mot
à ne pas couper dans une hbox.

Exemple :
Même quand il est en bout de ligne ce mot très long n'est pas
coupé CeMotTrèsLongNestPasCoupé.

Même quand il est en bout de ligne ce mot très long n'est pas
coupé \hbox{CeMotTrèsLongNestPasCoupé}.
%%%% fin exemple %%%%

* Pour empêcher LaTeX de couper les mots dans un paragraphe il
suffit de l'encadrer par les commandes \begin{sloppypar} et
\end{sloppypar}.

* Pour empêcher LaTeX de couper les mots dans tout un document,
il faut utiliser la commande \sloppy dans le préambule du
document. On peut également utiliser la déclaration :
\hyphenpenalty 10000.

* De manière globale, on peut aussi déclarer :
\DeclareFontFamily{T1}{cmr}{\hyphenchar\font=-1}

* Pour interdire la coupure de tous les mots commençant par une
majuscule, il faut utiliser : \uchyph=0 dans le préambule du
document.

* On peut également interdire la coupure des mots d'une langue
particulière dans un document multilingues en utilisant l'astuce
suivante : il suffit de créer un fichier de motifs de césures
vide, par exemple pour le russe
%%%% debut macro %%%%
%%% ruhyph.tex %%%
\patterns{}
\endinput
%%%%%%%%%%%%%%%%%%%
%%%% fin macro %%%%
et dans le fichier language.dat ajouter la ligne
russian ruhyph.tex

# 6.45 # Comment mettre en forme des équations chimiques ?
----------------------------------------------------------
* Le package 'ppchtex' disponible sur
ftp://ftp.fdn.org/pub/CTAN/macros/context/ permet d'écrire des formules chimiques.

* Sur Mac ou PC, il existe MDL qui est une version freeware
d'ISIS Draw qui permet de créer ses propres structures et de les
sauver sous format eps. Pour plus de détails, consulter
http://www.mdli.com/prod/ioffer.html.

* Il existe le package 'chemsym' disponible sur
ftp://ftp.fdn.org/pub/CTAN/macros/latex/contrib/supported/chemsym/.

* Le package 'XymTex' disponible sur
ftp://ftp.fdn.org/pub/CTAN/macros/latex/contrib/other/xymtex/ ou
ftp://ftp.fdn.org/pub/CTAN/macros/latex209/contrib/xymtex/ permet de définir des
structures chimiques.

Remarques (F. Jacquet) :
+ XyMTeX est incompatible avec le package french de B. Gaulle.
Pour utiliser les deux dans un même document, il suffit de
repasser en mode \english avant la macros puis \french après. Je
ne sais pas comment cela fonctionne pour les \caption mais en
théorie, ça devrait marcher !
AL: Vérifier la réalité de cette remarque par rapport au dernier french.
+ XyMTeX possède le gros défaut de ne pas pouvoir faire de
longues chaînes aliphatiques si on ne sait pas programmer le
nombre de points entre deux structures. Le plus simple dans ce
cas est donc l'emploi de Xfig (ou autre).
+ On ne peut pas imbriquer les formules, ce qui gène
considérablement son utilisation; en revanche, pour les
cholestérol l'ensemble est très puissant !

# 6.46 # Comment mettre en page un calendrier ?
-----------------------------------------------
* Le package 'termcal' disponible sur
ftp://ftp.fdn.org/pub/CTAN/macros/latex/contrib/supported/termcal/ permet de mettre
en page un calendrier. Il permet de préciser quels jours doivent
apparaître et d'insérer du texte soit régulièrement à certaines
dates soit à des dates particulières.

* Le package 'calendar ' disponible sur
ftp://ftp.fdn.org/pub/CTAN/macros/plain/contrib/ propose un ensemble de macros pour
mettre en page des calendriers.

* Un autre package 'calendar' est également disponible sur
ftp://ftp.fdn.org/pub/CTAN/macros/latex/contrib/supported/calendar/.

* Il existe également le package 'yplan97' disponible sur
ftp://ftp.fdn.org/pub/CTAN/macros/latex/contrib/other/yplan97/.

# 6.47 # Comment forcer un caption sur plusieurs lignes ?
---------------------------------------------------------
Pour forcer un retour à la ligne dans un caption, il faut
utiliser \caption[text1]{text2 \\\hspace{\linewidth} text3}.

Remarque : text1 est le texte qui apparaîtra dans la liste des
figures ou des tableaux.

# 6.48 # Comment générer des barres de modification dans un document ?
----------------------------------------------------------------------
Il suffit d'utiliser le package 'changebar' disponible sur
ftp://ftp.fdn.org/pub/CTAN/macros/latex/contrib/supported/changebar/.

Exemple :
dans le préambule :
\usepackage[outerbars]{changebar} % permet de positionner les
                                  % barres en marge externe
\setcounter{changebargrey}{20} % permet de fixer le niveau de
                               % gris des barres

et dans le document :
\begin{changebar}
   texte..
\end{changebar}

ou

ancien texte ancien texte ancien texte \cbstart nouveau
texte nouveau texte nouveau texte \cbend ancien texte ancien
texte ancien texte ancien texte
%%%% fin exemple %%%%

# 6.49 # Comment encadrer du texte verbatim ?
---------------------------------------------
* Le package 'moreverb' propose un environnement encadré.

Exemple : \begin{boxedverbatim} La commande \LaTeX permet
d'appeler le logo
   LaTeX.
\end{boxedverbatim}
%%%% fin exemple %%%%

# 6.50 # Comment écrire du texte en forme de losange ou autre ?
---------------------------------------------------------------
Le package 'shapepar' disponible sur
ftp://ftp.fdn.org/pub/CTAN/macros/latex/contrib/other/misc/ définit des
environnements losange, coeur, etc.

Exemple :
\diamondpar{mon paragraphe en forme de losange.}
%%%% fin exemple %%%%

# 6.51 # Comment isoler les flottants par un trait horizontal ?
---------------------------------------------------------------
* Pour que les flottants qui sont renvoyés en début
(respectivement en fin) de page soient isolés du texte par un
trait horizontal en dessous (respectivement au dessus), il faut
activer les options suivantes dans le préambule du document :
\let\topfigrule\hrule
\let\botfigrule\hrule

* Si leur comportement n'est pas satisfaisant, on peut le
redéfinir :
\newcommand{\topfigrule}{%
  \vspace*{3pt}\noindent\rule{\linewidth}{0.4pt}\vspace{-3.4pt}}
\newcommand{\botfigrule}{%
  \vspace*{3pt}\noindent\rule{\linewidth}{0.4pt}\vspace{-3.4pt}}

# 6.52 # Comment insérer un espace vertical dans une page ?
-----------------------------------------------------------
Il suffit d'utiliser la commande \vspace et de préciser en
argument la hauteur de l'espace voulu.

Exemple :
Texte avant.

\vspace{2cm}

Texte après.
%%%% fin exemple %%%%

Remarque : la commande \vspace* force l'insertion d'un espace
	   vertical même si ce dernier se situe sur un
	   changement de page.

# 6.53 # Comment insérer un espace horizontal dans un texte ?
-------------------------------------------------------------
De la même manière que pour les espaces verticaux, il existe la
commande \hspace pour insérer un espace horizontal dans un
texte.

Exemple :
blabla bla \hspace{3cm} bli bli bli
%%%% fin exemple %%%%

Remarque : la commande \hspace* force l'insertion d'un espace
	   même en début ou en fin de ligne.

# 6.54 # Comment définir un style de paragraphe ?
-------------------------------------------------
* Pour agir de manière globale (sur tout le document), les
paramètres de définition d'un paragraphe sont :
+ \parindent pour fixer la longueur des indentations
+ \parskip pour gérer l'espace entre les paragraphes.

Exemple :
\setlength{\parindent}{1cm}
\setlength{\parskip}{1ex plus 0.5ex minus 0.2ex}
%%%% fin exemple %%%%

* Pour forcer (supprimer) localement l'indentation d'un
paragraphe, il existe la commande \(no)indent.

# 6.55 # À quoi sert la commande \stretch ?
-------------------------------------------
La commande \stretch{x} permet d'insérer un espace élastique qui
va s'ajuster de manière à forcer l'occupation de toute une
ligne (en mode horizontal) ou de toute une page (en mode
vertical). Le paramètre x est appelé facteur d'élasticité, il
intervient dès que plusieurs commandes \stretch sont appelées
sur la même ligne.

Exemples :
+ bla\hspace{\stretch{1}} bla bla \hspace{\stretch{2}} bla bla
bla

+ un texte normal \vspace{\stretch{1}} et un texte qui finit de
remplir la ligne courante et qui se poursuit tout en bas de la
page sur la dernière ligne.
%%%% fin exemple %%%%

# 6.56 # Comment justifier un paragraphe à gauche ou à droite ?
---------------------------------------------------------------
* Les environnements flushleft et flushright permettent de
justifier à gauche ou à droite une portion de texte.

Exemple :
\begin{flushleft}
   Texte aligné à gauche. Texte aligné à gauche. Texte aligné à
   gauche. Texte aligné à gauche. Texte aligné à gauche. Texte
   aligné à gauche. Texte aligné à gauche. Texte aligné à
   gauche. Texte aligné à gauche. Texte aligné à gauche. Texte
   aligné à gauche.
\end{flushleft}
%%%% fin exemple %%%%

* Il existe également les commandes \raggedright et \raggedleft.

Exemple :
{\raggedleft
Colle à droite. Colle à droite. Colle à droite. Colle à droite.
Colle à droite. Colle à droite. Colle à droite.

Colle à droite. Colle à droite. Colle à droite. Colle à droite. Colle à
droite. Colle à droite. }
%%%% fin exemple %%%%

* Le package 'ragged2e' disponible sur
ftp://ftp.fdn.org/pub/CTAN/macros/latex/contrib/supported/ragged2e/ offre en plus des
commandes (\Centering, \RaggedLeft, et \RaggedRight) et des
environnements qui permettent de mieux gérer les césures.

# 6.57 # Comment mettre en page des citations ?
-----------------------------------------------
Il existe deux environnements de gestion des citations :
+ quote pour les citations courtes,
+ quotation pour les citations longues.

Exemple :
\begin{quote}
   L'environnement \texttt{quote} n'indente pas ces paragraphes
   par contre l'espace vertical entre ces derniers est supérieur
   à celui d'un texte standard.

   La preuve~!
\end{quote}

\begin{quotation}
   L'environnement \texttt{quotation} indente la première ligne
   de ses paragraphes et sépare ses paragraphes d'un espace
   vertical standard.

   La preuve~!

\end{quotation}

Dans les deux cas les marges droite et gauche sont plus
importantes que celles d'un texte standard.
%%%% fin exemple %%%%

# 6.58 # Comment insérer un texte sans que LaTeX le mette en forme ?
--------------------------------------------------------------------
L'environnement verbatim permet d'insérer un texte tel quel sans
que LaTeX ne le traite. Il permet notamment de faire apparaître
des commandes LaTeX.

Exemple :
\begin{verbatim}
   La commande \LaTeX permet d'imprimer le logo LaTeX.
\end{verbatim}
%%%% fin exemple %%%%

* La commande \verb?texte? permet de faire la même chose
localement. Le premier caractère ? permet de marquer le début du
mode verbatim et le second en marque la fin. Il peut être
remplacé par n'importe quel autre caractère sauf un espace ou *.

# 6.59 # Comment visualiser des espaces ?
-----------------------------------------
* L'environnement verbatim* et la commande \verb* permettent de
visualiser les espaces insérés dans un texte en les remplaçant
par un caractère spécial en forme de u.

Exemple :
Je mange \verb*?   ? pomme.
%%%% fin exemple %%%%

* LaTeX propose la commande \textvisiblespace.

* En TeX, c'est le caractère 32 de la fonte cmtt, qui est
défini dans le source du TeXBook par :
\def\]{\leavevmode\hbox{\tt\char`\ }} % visible space

Vous pouvez donc définir:
\def\vs{\leavevmode\hbox{\tt\char`\ }} % visible space
et \vs donnera ce fameux caractère.

# 6.60 # Comment insérer du code LaTeX dans un document LaTeX ?
---------------------------------------------------------------
* Le package 'example' offre un environnement example qui permet en
ne tapant qu'une seule fois le code d'avoir côte à côte le code
LaTeX et son résultat après compilation.

Exemple :
\begin{example}
   La commande \LaTeX permet d'appeler le logo LaTeX.
\end{example}
%%%% fin exemple %%%%

*  La solution la plus classique consiste à faire :
\hbox{\vtop{\hsize 0.5\hsize le permier texte}\vtop{\hsize
0.5\hsize{\begin{verbatim} le même texte \end{verbatim}}}}

# 6.61 # Comment écrire du texte en couleurs ?
----------------------------------------------
* Voir à ce sujet le Cahier GUTenberg numéro 16 (février 1994,
entièrement consacré à ce problème) et l'article de M. Goossens
et M. Jouhet dans Cahier GUTenberg 21 (juin 1995, pages 30-52),
accessible à l'URL :
http://www.gutenberg.eu.org/publications/.

* Le package 'color' est disponible sur
ftp://ftp.fdn.org/pub/CTAN/macros/latex/packages/graphics/. Il permet de coloriser le
texte ou le fond du document.

Exemple :
\textcolor{red}{Important}
%%%% fin exemple %%%%

* pstricks disponible sur ftp://ftp.fdn.org/pub/CTAN/graphics/pstricks/ est un
ensemble de macros PostScript compatibles avec Plain
TeX, LaTeX, AmSTeX, and AmS-LaTeX. Il comprend notamment
des macros pour la colorisation, la gestion des graphiques, le
dessin de camembert, d'arbres, etc.

* Voir également la question 6.25 : Comment griser le fond
(background) d'un paragraphe ?

# 6.62 # Comment construire un arbre ?
--------------------------------------
* Le package 'qobitree', est disponible sur
ftp://ftp.fdn.org/pub/CTAN/macros/latex/contrib/other/qobitree/.

* Le package 'treesvr' est également disponible sur
ftp://ftp.fdn.org/pub/CTAN/macros/latex/contrib/supported/treesvr/.

Exemple :
\begin{picture}(100,100)
\setlength\unitlength{2mm}
\branchlabels ABC         % 012 is the default
\root(2,10)        0.     % root at absolute coordinate (2,10)
                          % its (internally used) label is 0
                          % the space before the 0 obligatory
\branch2{16}       0:1,2. % node 0 has children 1 and 2
		   	  % the text "16" is written above it
                          % space optional, :,. obligatory
\leaf{4}{$u_1$}    1.     % node 1 is a leaf
                          % "4" written above, "$u_1$" at right
\branch2{12}       2:3,7. % branch to node 3 goes up labeled A
  \tbranch2{9}     3:4,5,6.
    \leaf{4}{$u_3$}4.
    \leaf{3}{$u_4$}5.
    \leaf{2}{$u_5$}6.
  \leaf{3}{$u_2$}  7.
\end{picture}
%%%% fin exemple %%%%

* Le package 'treetex' disponible sur
ftp://ftp.fdn.org/pub/CTAN/macros/latex209/contrib/trees/tree_tex/ permet de metttre en
page des arbres sous LaTeX.

* Les packages 'epic', 'eepic', 'ecltree' permettent de créer des
arbres (les trois packages sont nécessaires pour une
utilisation avec Latex2e pour garder la compatibilité avec
Latex2.09).

Exemple :
\documentstyle[epic,eepic,ecltree]{article}
\begin{document}

\begin{bundle}{racine}
\chunk{feuille 1}
\chunk{feuille 2}
\drawwith{\dottedline{3}}
\chunk{feuille 3}
\drawwith{\drawline}
\chunk{feuille 4}
\end{bundle}

\end{document}
%%%% fin exemple %%%%

# 6.63 # Comment mettre en page un poème ?
------------------------------------------
* Il existe l'environnement verse. Il gère les marges, les
retours à la ligne dans une strophe se font par \\ et les
strophes sont séparées par des lignes blanches.

Exemple:
\begin{verse}
   J'ai un poney gris, \\
   Qui galope à travers la prairie.

   Grignote, grignote dans ma main, \\
   La carotte rousse du jardin.

   Mes cousins ont un poney blanc, \\
   Qui parcourt chemins et champs.
\end{verse}
%%%% fin exemple %%%%

* Ci-dessous un exemple de style de JHB Nijhof
(nijhojhb@aston.ac.uk) :

Exemple :
\documentstyle[a4,12pt]{article}

\def\testline{\par\noindent\hbox to 0pt{\hss*\hss}\hfill
\hbox to 0pt{\hss*\hss}\hfill\hbox to 0pt{\hss*\hss}\par}
\def\centerstar{\par\medskip\noindent\hbox to\hsize%
{\hss*\hss}\par\medskip}

\newbox\poembox
\newbox\widebox
\newdimen\centerx
\newcount\linecount
\newdimen\poemleftmargin
\def\newpoem{\setbox0=\box\poembox \setbox0=\box\widebox
\linecount=0} % empty boxes
\newpoem % probably not necessary
\def\poemline#1{\setbox0=\hbox{\strut #1}%
 \setbox\poembox=\vbox{\unvbox\poembox\copy0}%
 \setbox\widebox=\hbox{\unhbox\widebox\copy0}%
 \advance \linecount 1}
\def\setpoem{% you'll want a clearpage here
\centerx=\wd\widebox \divide\centerx\linecount \divide\centerx 2%
%now centerx is the x of the center of gravity
\poemleftmargin=0.5\hsize \advance \poemleftmargin-\centerx
\noindent\kern\poemleftmargin\box\poembox
\par
\newpoem}

\begin{document}

\testline % for viewing the margins/ middle
\poemline{Rozen verwelken}
\poemline{schepen vergaan}
\poemline{maar onze liefde}
\poemline{zal blijven bestaan}
\setpoem

\centerstar

\poemline{Rozen verwelken}
\poemline{schepen vergaan}
\poemline{maar onze liefde zal blijven bestaan}
\setpoem

\centerstar

\poemline{Rozen verwelken, schepen vergaan}
\poemline{maar onze liefde zal blijven bestaan}
\setpoem

\centerstar

\poemline{Rozen verwelken}
\poemline{\qquad schepen vergaan}
\poemline{maar onze liefde}
\poemline{\qquad zal blijven bestaan}
\setpoem

\centerstar

\end{document}
%%%% fin exemple %%%%

# 6.64 # Comment aligner des paragraphes ?
------------------------------------------
On peut utiliser la commande \parindent.

Exemple :
{\settowidth{\parindent}
{Notes~:\ }

\makebox[0pt][r]
{Notes~:\ }La première note pour dire que ...

La seconde pour préciser que ...
}
%%%% fin exemple %%%%

# 6.65 # Comment construire une liste d'éléments ?
--------------------------------------------------
Il existe trois environnements de liste par défaut :
+ l'environnement itemize
+ l'environnement description
+ l'environnement enumerate

Exemples :
\begin{itemize}
   \item un élément
   \item un autre élément
\end{itemize}

\begin{description}
   \item[Genre] Le genre peut être féminin ou masculin
   \item[Nombre] Le nombre peut être singulier ou pluriel
\end{description}

\begin{enumerate}
   \item premier élément
   \item deuxième élément
\end{enumerate}
%%%% fin exemple %%%%

# 6.66 # Comment faire une note dans une marge ?
------------------------------------------------
On peut utiliser la commande \marginpar{note marge}.

Exemple :
La valeur du paramètre temps est fixée à
12~minutes\marginpar{AC}. En tenant compte de cette hypothèse
les résultats obtenus sont les suivants~:

~\ldots
%%%% fin exemple %%%%

Remarque : Pour inverser les notes dans les marges
	   (droite/gauche) il suffit de mettre dans le
	   préambule : \reversemarginpar.

# 6.67 # Comment faire une page de garde ?
------------------------------------------
Les commandes permettant de définir une page de garde sont :
+ \title pour un titre
+ \author pour le ou les auteurs (dans ce dernier cas leurs noms
sont séparés par la commande \and)
+ \date pour la date
+ \thanks utilisée dans l'une des commandes ci-dessus permet de
faire référence à une note de bas de page.

La page de garde est ensuite générée par la commande \maketitle
appelée dans le corps du document (généralement juste après la
balise \begin{document}).

Exemple :
\documentclass{report}
\usepackage{french}

\newlength{\larg}
\setlength{\larg}{14.5cm}

\title{
{\rule{\larg}{1mm}}\vspace{7mm}
\begin{tabular}{p{4cm} r}
   & {\Huge {\bf {FAQ} \LaTeX{} française}} \\
   & \\
   & {\huge Pour débutants et confirmés}
\end{tabular}\\
\vspace{2mm}
{\rule{\larg}{1mm}}
\vspace{2mm} \\
\begin{tabular}{p{11cm} r}
   & {\large \bf Version 2.0} \\
   & {\large  \today}
\end{tabular}\\
\vspace{5.5cm}
}
\author{\begin{tabular}{p{13.7cm}}
Marie-Paule Kluth
\end{tabular}\\
\hline }
\date{}

\begin{document}
\maketitle

Voici mon document.

\end{document}
%%%% fin exemple %%%%

# 6.68 # Comment utiliser \thanks dans chaque titre de chapitre ?
-----------------------------------------------------------------
Pour associer des remerciements dans chaque chapitre d'un thèse,
par exemple, il faut définir sa propre macro à base de
\footnote.

Exemple (R. Fairbairns) :
\documentclass{report}
\usepackage{french}

\newcommand\thankschapter[2]{%
   % arg 1 is chapter title
   % arg 2 is `thanks' text
   \edef\savefootnote{\thefootnote}
   \renewcommand\thefootnote{\fnsymbol{footnote}}
   \chapter[#1]{#1\footnote[1]{#2}}
   \renewcommand\thefootnote{\arabic{footnote}}
   \setcounter{footnote}{\savefootnote}
}
\begin{document}
\chapter{Préface.}{\textit{Un grand merci à M. Nigaudon pour
avoir accepter de préfacer ce livre.\\}}

Ce livre traitant de \LaTeX,~\ldots

\end{document}
%%%% fin exemple %%%%

# 6.69 # Comment mettre un résumé et un abstract dans un document ?
-------------------------------------------------------------------
* Le package 'french' propose les environnements resume et
abstract.

Exemple (textes sur la même page) :
\documentclass{article}
\usepackage{french}
\pagestyle{empty}
\begin{document}
\begin{resume} résumé en français \end{resume}
   \english
\begin{abstract} the same in english \end{abstract}
   \french

   Cet article~\ldots
\end{document}
%%%% fin exemple %%%%

Remarque : \resume n'est qu'un alias de \abstract.

Exemple (chaque résumé est sur une page séparée) :
\begin{english}
   \begin{abstract}
      english text
   \end{abstract}
\end{english}

\begin{french}
   \begin{abstract}
      Texte français
   \end{abstract}
\end{french}
%%%% fin exemple %%%%

* Le package 'babel' peut également être utilisé.

Exemple :
\usepackage[francais,english]{babel}
...
\selectlanguage{francais}
resume : je parle français
....
\selectlanguage{english}
abstract : I speak english
....
%%%% fin exemple %%%%

* Autre solution qui ne permet toutefois pas d'avoir les deux
textes sur la même page :

%%%% debut macro %%%%
\renewcommand{\abstractname}{Résumé}
\begin{abstract} résumé en français \end{abstract}
\renewcommand{\abstractname}{Abstract}
\begin{abstract} the same in english \end{abstract}
%%%% fin macro %%%%

# 6.70 # Comment définir de nouvelles couleurs ?
------------------------------------------------
Il existe deux repères pour cela :
+ cyan, magent, yellow, black et la commande :
\definecolor{nouveau_nom}{cmyk}{w,x,y,z}
\newcmykcolor{le_nom_de_la_couleur}{w x y z}
avec w,x,y,z entre 0.0 et 1.0

+ red,green, blue et la commande :
\definecolor{nouveau_nom}{rgb}{a,b,c}
ou : \newrgbcolor{le_nom_de_la_couleur}{a b c}
avec a,b,c entre 0.0 et 1.0

+ il existe également :
\newhsbcolor{le_nom_de_la_couleur}{num1 num2 num3}
qui permet de jouer sur le << hue >>, la saturation et la
luminosité.

# 6.71 # Comment placer des figures face à face en recto-verso ?
----------------------------------------------------------------
Dans un document recto-verso, pour s'assurer qu'une figure
apparaisse sur la page de gauche et une autre sur la page de
droite en face, il faut utiliser le package 'dpfloat' disponible
sur: http://weber.u.washington.edu/~fox/tex/dpfloat.html.

# 6.72 # Quelles sont les différences entre a4paper et letterpaper ?
--------------------------------------------------------------------
L'option a4paper exécute :
  \setlength\paperheight {297mm}
  \setlength\paperwidth  {210mm}

L'option par défaut est letterpaper qui fait
  \setlength\paperheight {11in}
  \setlength\paperwidth  {8.5in}

Remarque (J. Kanze) : Il faut remarquer que la différence n'est
	   pas énorme : les défauts en mm sont 279x216. En fait,
	   sur le petit test que j'ai fait, les coupures de
	   lignes se trouvaient exactement aux mêmes endroits --
	   en revanche, chaque page contenait bien quelques
	   lignes de plus avec a4paper.

# 6.73 # Comment couper une chaîne de caractères ?
--------------------------------------------------
* Pour forcer la coupure d'une chaîne de caractères, il faut
utiliser la commande \- à l'endroit où la chaîne doit être
coupée.

Exemple :
Je décide que le dernier mot de cette ligne soit coupé en deux
ici:~ do\-cumentation. N'est-ce pas beau~?
%%%% fin exemple %%%%

* Pour indiquer à LaTeX comment couper certain mots, on peut
utiliser la commande \hyphenation.

Exemple :
\hyphenation{do-cu-ment}
Je décide que le dernier mot de cette ligne soit coupé en deux
ici:~ documentation. N'est ce pas beau~?
%%%% fin exemple %%%%

# 6.74 # Comment modifier l'espace entre une figure et sa légende ?
-------------------------------------------------------------------
Les deux variables qui définissent respectivement l'espace avant
une légende et l'espace après sont : \abovecaptionskip et
\belowcaptionskip.

Exemple de redimensionnement :
\setlength\abovecaptionskip{0.25ex}
%%%% fin exemple %%%%

--
Benjamin Bayart
bayartb@edgard.fdn.fr